CIC Practice Questions

अब Quizwiz के साथ अपने होमवर्क और परीक्षाओं को एस करें!

The p value in statistical test results indicates: a. causation b. the probability of having committed a type I error c. the probability of having committed a type II error d. the probability of data being accurate and valid

B A type I error occurs when one rejects the null hypothesis (Ho) when it is true. This is also called a false-positive result (as we incorrectly conclude that the research hypothesis is true when in fact it is not). The p value or calculated probability is the estimated probability of rejecting the null hypothesis of a study question when that hypothesis is true

An employee who is not immune to varicella-zoster was exposed to a patient with active chickenpox. How long must the employee remain on work restrictions? a. Until evaluated by a physician b. From day 10 after exposure to day 21 after exposure c. No work restriction is necessary if no signs and symptoms are present d. At the discretion of the hospital infectious disease physician

B According to the ACIP guidelines, a nonimmune health care worker who has direct contact with a patient with varicella zoster should be excluded from work duty from the 10th day after the first day of exposure through the 21st days after last exposure

A food service worker is diagnosed with Hepatitis A. How long should this employee be on work restrictions? a. Until 14 days after symptoms resolve b. Until 7 days after onset of jaundice c. Until 14 days after onset of jaundice d. Until 10 days after symptoms resolve

B According to the ACP, food service workers who are diagnosed with Hepatitis A must be restricted from food handling until 7 days after the onset of jaundice

What percentage of Creutzfeldt-Jakob disease (CJD) cases are sporadic (vs familial)? a. 65 percent b. 85 percent c. 15 percent d. 1 percent

B CJD occurs naturally in either of two forms: the sporadic type (occurring at a rate of 1 case per 1 million population) and the familial type due to a genetic mutation that can be passed from generation to generation and has been documented in geographical clusters in various parts of the world. The percentage of sporadic CJD is 85 percent. Sporadic CJD has no gender restrictions and occurs at a mean onset of 50 to 70 years of age

During an influenza outbreak in an acute care setting, all the following measures should be implemented except: a) perform rapid influenza virus testing of patients and personnel with recent onset of symptoms suggestive of influenza b) implement airborne precautions for all patients with suspected or confirmed influenza c) restrict staff movement from areas of the facility having outbreaks d) restrict or limit elective medical and surgical admissions

B During an influenza outbreak in an acute care setting, the following measures should be taken to protect patients and staff and to reduce the risk of health care-associated influenza transmission: -perform rapid influenza virus testing of patients and personnel with recent onset of symptoms suggestive of influenza -implement droplet precautions for all patients with suspected or confirmed influenza -separate suspected or confirmed influenza patients from asymptomatic patients -restrict staff movement from areas of the facility having outbreaks -immunize unvaccinated patients and health care personnel with current recommended, available influenza vaccine -administer influenza antiviral chemoprophylaxis and treatment to patients and health care personnel according to current recommendations -consider antiviral chemoprophylaxis for all health care personnel, regardless of their vaccination status, if the health department determines the outbreak is caused by a variant of influenza virus that is suboptimal match with the vaccine -limit or stop elective medical and surgical admissions -restrict cardiovascular and pulmonary surgery to emergency cases during influenza outbreaks, especially those characterized by high attack rates and severe illness, in the community or acute care facility

Which of the following is recommended for use when inserting a central venous catheter or a peripherally inserted catheter? a. A cap b. A clean gown c. Nonsterile gloves d. Small drapes

A according to the CDC guidelines for the prevention of intravascular catheter-related infections, maximal sterile barrier precautions must be used for the insertion of CVCs, PICCs, or guidewire exchanges. This includes the use of a cap, mask, sterile gown, sterile gloves, and a sterile full-body drape for the insertion

What is not considered a risk factor for young girls to develop a UTI? a. bubble baths and prolonged soaking in the bathtub b. direction of wiping with toilet pater (from back to front) c. excessive holding of urine d. delayed bladder emptying

B approximately 2.2% of girls under the age of 2 develop a UTI. risk factors for young girls include: -history of maternal UTI -family history of vesicoureteral reflux -history of dysfunctional voiding patterns -constipation although often reported as a cause, direction of wiping with toilet paper is not a risk factor. identified risk factors include bubble baths and prolonged soaking in the bathtub, excessive holding of urine, and delayed bladder emptying

Which of the following is not an advantage of a case-control study? a. It takes less time than a cohort study b. It requires fewer subjects c. The data are more accurate because it is prospective d. It is generally considered less expensive

C

Which of the following steps are not included in hypothesis testing? a. State the null and alternative hypotheses b. set the significance level c. eliminate outliers d. compare the probability value to the significance level

C A common use of statistics is hypothesis testing. It is a statement of expected results. Hypothesis testing uses the distribution of a known area in the normal curve. It estimates the likelihood (probability) that a result did not occur by chance. Steps to hypothesis testing include: -state the research question -specify the null and alternative hypotheses -calculate test statistic -compute probability of test statistic or rejection region -state conclusions outliers are observations that deviate from all others significantly. they may occur by accident, or they may be the result of measurement errors. analysis and dealing with outliers is an important component of statistical analysis. sometimes careful analysis of outliers, their removal, or weighting down can change the conclusions considerably. outliers should be investigated to determine the optimal method of analysis

Which type of isolation always requires a private room? a. Contact precautions b. Standard precautions c. Airborne precautions d. Droplet precautions

C According the CDC isolation guidelines, private rooms are not required for contact and droplet precautions (though they are preferred). Patients on contact or droplet isolation with the same disease may share a room if necessary. Patients on Airborne Isolation require a private room

The IP wishes to know the proportion of a disease that could be prevented by eliminating the exposure in the entire study population. in order to determine this information, the IP will need to calculate the: a. attributable fraction b. attributable risk c. population attributable risk percent d. negative predictive value

C attributable risk percent (ARP) is a calculation that can be derived from the attributable risk, which is the difference in rate of a condition between an exposed population and an unexposed population. ARP gives the proportion of cases attributable (and avoidable) to this exposure in relation to all cases. It can be calculated as: (relative risk - 1) / relative risk

Indirect and direct causes of disease may form a complex network of events that determine the level of disease in a community. Which of the following is the term for this interrelation of events? a. applied epidemiology b. iceberg phenomenon c. causal web d. dendrogram

C causation is an essential concept in epidemiology. the web of causation refers to the interrelationship of multiple factors that contribute to the occurrence of a disease

The CDC and WHO guidelines for hand hygiene recommend the use of an alcohol-based hand rub in all of the following situations except: a. After direct patient contact b. Before donning sterile gloves c. When hands are visibly soiled d. When moving from a contaminated body site to a clean body site during patient care

C hand hygiene is a critical component of patient and employee safety. Use of alcohol-based hand rubs has increased adherence of health care personnel to recommended hand hygiene policies and have been associated with reduced HAI rates. However, when hands are heavily soiled or greasy, hand sanitizers may not work. Hand washing with soap and water is recommended in such circumstances.

All of the following may be indications of a heating, ventilation, and air conditioning (HVAC) malfunction except: a. an increase in the postoperative surgical site infection (SSI) rates b. a single case of aspergillosis in a severely immunosuppressed patient c. healthcare-associated varicella infections d. an outbreak of ventilator-associated acinetobacter infections in the ICU

*** This needs up be updated. Correct answer is D. Look in APIC text chapter 114- Heating, Ventilation, and Air conditioning***** originally put C HVAC systems can be a source of exposure and infection caused by airborne infectious agents. aspergillus, rhizopus, and penicillium as well as fungi may be hazardous to immunosuppressed patients. can get into water. can be spread through air into surgery rooms and ventilators. varicella is chicken pox, spread via airborne but most people are vaccinated against. can also pass via lesions and rashs.

Which of the following organisms have been associated with the transmission of infections after body piercing? 1. Atypical Mycobacterium species 2. Staphylococcus species 3. Pseudomonas species 4. Haemophilus species

*****This needs to be updated. Answer is 1 2 and 3, got incorrect. Look for answer in APIC Chapter 123-Body piercings, tattoos, and electrolysis***** 2 since staph lives on skin and in body atypical mycobacteria (nontuberculosis) are among the most commonly reported waterborne pathogens; can get into potable water. can get in hospital equipment. staphylococcus generally divided into two groups (coagulase + (like staph aureus) and coagulase - (like s epidermidis). often part of normal microbiota. common causes of HAI's pseudomonas are gram negative bacilli that have many reservoirs in hospital settings and cause opportunistic infections. aerobic and moist environments. tolerant to many conditions and require minimal nutrition. innate antibiotic resistance. any wet healthcare environment could harbor haemophilus include a wide variety of species and infections. most common H influenzae can cause meningitis, epiglottitis, bacteremia, cellulitis

What is the attributable risk between exposed and unexposed population in the following table? 2x2 table where cell A (disease, unexposed) is 9. B (disease, exposed) is 17. C (no disease, unexposed) is 7. D is 5

0.21 attributable risk is the difference in rate of a condition between an exposed population and an unexposed population. the formula for attributable risk is as follows: IE-IU=AR, where IE=incidence in the exposed and IU=incidence in the unexposed. IE is calculated by diving the number of exposed people who get the disease by the total number who are exposed. Similarly, the IU is calculated by dividing the number of unexposed people who get the disease by the total number who are not exposed. In this table, AR = 0.77-0.56 = 0.21

Guidelines for transporting specimens include: 1) transport within 2 hours of collecting a specimen 2) transport in leakproof specimen containers and sealable leakproof bags 3) transport specimen in the syringe used to collect it 4) refrigerate all specimens prior to transport

1, 2 The appropriate selection, collection, and transport of specimens to the diagnostic microbio lab are essential parts in accurate identification of microorganisms that cause infections that affect patient care and infection prevention. transportation guidelines include: -all specimens must be promptly transported to the lab, preferably within 2 hours of collection. delays or exposure to temp extremes can compromise test results. -specimens should be transported in a container designed to ensure survival of suspected agents. never refrigerate spinal fluid, genital, eye, or internal ear specimens because these samples may contain microorganisms sensitive to temp extremes -materials for transport must be labeled properly, packaged, and protected during transport. a transport medium can be used to preserve the viability of microorganisms in clinical samples. -use leakproof specimen containers and transport them in sealable, leakproof plastic bags -never transport syringes with needles attached -laboratories must have enforceable criteria for rejection of unsuitable specimens

Dialysis staff are most at risk for exposure to bloodborne pathogens during: 1. Initiation and termination of dialysis 2. Reprocessing, cleaning, and disinfection procedures 3. Medication administration 4. Vascular access hemorrhage

1, 2 staff members must follow standard precautions when exposure to blood or other potentially infectious materials is anticipated or likely. times during which exposure is most likely to occur include initiation and termination of dialysis and during reprocessing, cleaning, or disinfection procedures.

The institute for healthcare improvement uses the model for improvement-a two-part model designed to accelerate improvement for health care processes and outcomes. What are the key component areas of this model? 1) setting aims, establishing measures, selecting changes 2) plan-do-study-act 3) contemplation, action, termination 4) perceived seriousness and cues to action

1, 2 the first part of the model for improvement includes setting aims (asking what are we trying to accomplish), establishing measures (how to know that the change leads to an improvement), and selecting change that will make an improvement. The second part of the model for improvement involves testing the selected changes in a plan-do-study-act cycle. small-scale testing is following by refinement and more testing until the changes are ready to be rolled out on a larger scale

The IP has been benchmarking data to other facilities performing similar activities for a period of time. the IP should analyze the entire process to ensure that which of the following conditions are met? 1) standardized definitions are used consistently 2) overall rates are used to accurately track trends over time 3) adequate training of personnel to collect, store, manage, and analyze data 4) data are calculated using the same methodology as nationally validated system

1, 2, 3, 4 to accurately trend surveillance data over time within a facility or compare rates between facilities, surveillance criteria (i.e., case definitions) must be consistently used to determine the presence of an HAI, occurrence of an event, or compliance with a process. rates, rather than raw numbers, must be used to accurately track trends over time. personnel who are responsible for collecting and managing surveillance data must have adequate training in reviewing medical records, interpreting clinical notes, applying standardized criteria for identifying cases, using appropriate statistical and risk adjustment methods, and using computer tools and technology (especially electronic records, spreadsheets, and databases) to collect, store, manage, and analyze data. whenever possible, data should be expressed as rates or ratios that are calculated using the same methodology as a nationally validated surveillance system. this allows an organization to compare its rates with another organization or a recognized benchmark.

The management of an infected surgical site includes the following foundational principles: 1. Open and drain the incision 2. Debride fibrous debris and necrotic soft tissue 3. Replace hardware 4. Implement antimicrobial management as needed 5. Manage the open wound

1, 2, 4, 5 each infected wound has a unique patient profile, unique characteristics of the operation, and potentially unique bacteriological features. the diversity of variables in the infected surgical site has resulted in a diverse number of options that are employed in management. the foundational principles in the management of the infected surgical site are 1) open and drain the incision 2) debride fibrinous debris and necrotic soft tissue 3) remove foreign bodies 4) implement antimicrobial management as needed and 5) manage the open wound

Important considerations regarding blood culture specimens include: 1) Collect prior to the initiation of antimicrobial therapy 2) Collect from a central venous catheter whenever possible 3) Ensure that the volume of the specimen collected is sufficient 4) Culture of specific sites is not recommended for surveillance a. 1, 3 b. 2, 4 c. 1, 4 d. 3, 4

A

Patients with cell-mediated immunity dysfunction are susceptible to infections attributed to pathogenic intracellular bacteria. Examples of these organisms include: 1. Salmonella typhi 2. Bacteroides fragilis 3. Listeria monocytogenes 4. Staphylococcus aureus

1, 3 As a host becomes progressively more immunocompromised, progressively fewer organisms are required to infect the host. Thus, patients with major immune defects are subject to a larger number and greater variety of infectious diseases. The most common opportunistic bacterial infections associated with cell-mediated immunity dysfunction are primarily intracellular pathogens that include: -Listeria monocytogenes -Salmonella spp. -Mycobacterium spp., including M. tuberculosis -Nocardia (N. asteroides, others) -Legionella pneumophila, other species Legionella -Rhodococcus equi -Pseudomonas pseudomallei

Which of the following are true about a frequency polygon: 1) It is useful for showing two sets of data on a single graph 2) Uses bars on the x axis 3) Uses connecting lines and data points 4) Depicts the percentage of the total that each datapoint represents.

1, 3 graphic or pictorial statistics present the numerical data that have been collected in graphs or charts, creating a picture of the data. a frequency polygon is a graph of a frequency distribution with values of the variable on the x axis and the number of observations on the y axis; data points are plotted at the midpoints of the intervals and are connected with a straight line. example on #36 in class notes

A patient who underwent intraocular surgery was diagnosed with a noninfectious endopthalmitis after the procedure. The IP initiates an investigation to identify the possible cause. Which of the following factors should be considered? 1. Improper handling, cleaning, and rinsing of the instruments 2. improper labeling of the solutions 3. gloves and powder 4. prophylactic antibiotics administered 2 hours before the procedures

1, 3 noninfectious endophthalmitis is an adverse event with several etiologies, including retained lens material and other introduced toxic substances. most often associated with TASS, an acute, rapid onset of sterile anterior segment inflammation that mimics infectious endophthalmitis, most commonly occurs after cataract surgery. cases are associated with breaches of handling, cleaning, and disinfecting of surgical instruments; introduction of contaminated solutions; contaminated intraocular lenses; toxic medications during surgery; and powder from gloves and irritants left of instruments

Which of the following rules should be followed when collecting a stool sample for C. difficile testing? 1. stool sample should be freshly passed within 1-2 hours 2. 10-20mL. of formed stool should be collected 3. stool should be passed into a clean, dry container 4. specimens should be obtained before antimicrobial agents have been administered

1, 3 CDI is an infectious bacterial diarrhea that occurs almost invariably in patients who have taken antibiotics recently (usually within 2 months); one of the most common and costly HAIs. Patients who produce formed stool specimens are not likely to be infected with C. difficile. testing on formed fecal samples represent an unnecessary use of lab resources and may result in false-positive.

The chi-square test can be used: 1) to evaluate the effect of a variable on outcomes 2) to analyze continuous data 3) to calculate an odds ratio or relative risk 4) if each cell of the table is greater than 5

1, 3, 4 Chi-square tests (X^2) can be used to test the association between two classifications of a set of counts or frequencies (discrete data). This data are commonly displayed as a contingency table or 2x2 table where tows represent one variable and columns represent the other. The null hypothesis is that there is no association between the two variables. Row and column totals (marginal totals) are used to predict what count would be expected for each cell if the null hypothesis were true. A test statistic is calculated from the observed and expected frequencies. The larger the test statistic (for given df) the more likely there is to be a statistically significant association between the two variables. Chi=square tests are used for medium to large samples. the fisher's exact test is used in place of the X^2 when the sample size number is less than 20 or any one cell in the table is less than 5.

Numerous outbreaks of infections have been attributed to unsafe injection practices. The IP designs an educational program to review safe injection practices with all nursing staff. These practices include: 1) Use single-dose vials whenever possible and avoid using multidose vials 2) Discard saline bags used for (IV) flushes for multiple patients after 1 hour 3) Enter medication vials with a new needle and syringe, even on the same patient 4) Use needles and syringes for only one patient

1, 3, 4 Since 1999, more than 125000 patients in the US have been notified of potential exposure to Hepatitis B virus, Hepatitis C virus, and HIV due to unsafe injection practices. Many of these incidents involved health care providers reusing syringes, resulting in contamination of medication vials or containers, which were then used on subsequent patients. Injection safety and other basic infection control practices are central to patient safety. Safe injection practices include: -never administer medications from the same syringe to more than one patient, even if the needle is changes -do not enter a vial with a used syringe or needle -medications packaged as single-use vials must never be used for more than one patient -medications packaged as multiuse vials should be assigned to a single patient whenever possible -bags or bottles of IV solution must not be used as a common source of supply for more than one patient -absolute adherence to proper infection control practices must be maintained during the preparation and administration of injected medications

Human factor limitations that contribute to errors include: 1) overdependence on multitasking skills 2) permanent night shifts 3) limited memory capacity 4) stress, fatigue, and sensory overload

1, 3, 4 human factors refer to environmental, organizational and job factors, and human and individual characteristics, which influence behavior at work in a way that can affect health and safety. Human factor limitations that contribute to errors include: -limited memory capacity: five to seven pieces of information are typical for short-term memory -negative effects of stress and associated cognitive tunnel vision used to compensate and focus in highly intense situations -negative influence of fatigue and sensory overload -overdependence on multitasking skills of staff in complex work environments

For which of the following procedure(s) is the surveillance period for deep incisional or organ/space SSI 90 days? 1) cesarean section 2) craniotomy 3) coronary artery bypass graft 4) laminectomy

2, 3 according the the CDC, SSI surveillance definitions, postoperative surveillance for deep incisional or organ/space SSIs should be conducted for 90 days on craniotomy and coronary artery bypass procedures. Superficial incisional SSIs are only followed for a 30-day period for all procedure types

An example of an obligate intracellular parasitic bacterium would be an organism responsible for: 1. Hepatitis 2. Q fever 3. Malaria 4. Epidemic typhus

2, 4 intracellular parasites are parasitic microorganisms that are capable of growing and reproducing inside the cells of a host. obligate intracellular parasites cannot reproduce outside their host cell, meaning that the parasite's reproduction is entirely reliant on intracellular resources. All viruses are obligate intracellular parasites. Obligate intracellular parasitic bacteria include Chlamydia, Rickettsia, Coxiella, and certain species of Mycobacterium

Which of the following are acceptable methods for follow-up testing among health care personnel with unprotected exposure to TB? 1. QuantiFERON-TB Gold testing (QFT-G) of sputum at the time of exposure and 12 weeks after exposure 2. QFT-G testing of blood at the time of exposure and 12 weeks after exposure 3. TST via tine tests at the time of exposure and 12 weeks after exposure 4. TST via the intradermal method at the time of exposure and 12 weeks after exposure 5. Chest radiograph for personnel with prior positive TST or QFT-G results 6. Chest radiograph for symptomatic personnel with positive TST or QFT-G results

2, 4, 6 QFT-G is a blood assay. Intradermal rather than tine testing methods should be used for TST. Testing should be administered at the time of exposure and repeated at 12-week postexposure to look for possible converters. Chest radiographs are performed only on those with prior positive screening results who are currently symptomatic

In a case control study, the association between obesity and C diff was examined. the table below provides the results. which of the following odds ratios is correct? a 2x2 table where cell a is 55, b is 30, c is 45, and d is 70.

2.85 The OR is the prob of having a particular risk factor if a condition or disease is present divided by the probability of having the risk factor if the disease or condition is not present. it is used for all types of studies with nominal data, but it is used to mostly for retrospective and cross sectional studies. the OR is sometimes called the cross-product ratio or relative odds (55x70) / (30x45) = 2.85

An IP is preparing the quarterly report for the infection control committee. what information will be needed for calculate a CLABSI rate for the ICU? 1) the total number of patients in the unit for the time period 2) the total number of central line catheters for the time period 3) the number of patients who had bloodstream infections identified 4) the number of device days for the time period

3, 4 the numerator would be the number of patients who had bloodstream infections identified and who had a central line during the time period. the denominator would be the number of device days for the time period (counted at the same time every day, count the number of patients with one or more central lines) basic formula for all rates: rate = x/y * k where: x= the numerator, which equals the number of time the event (e.g. infections) has occurred during a specified time interval y= the denominator, which equals a population (eg number of patients at risk) from which those experiencing the event were derived during the same time interval k= a constant used to transform the result of division into a uniform quantity so that it can be compared with other, similar quantities. a whole number (fractions are inconvenient) such as 100, 1000, 10000, or 100000 is usually used (selection of k is usually made so that the smallest rate calculated has at least one digit to the left of the decimal point) or is determined by accepted practice (the magnitude of numerator compared with denominator)

The Director of the Operating Room (OR) requests that the OR surfaces be routinely environmentally cultured. The IP's best response should be: 1. A schedule for routine culturing of the OR should be arranged so that each room is cultured as a set interval 2. Routine culturing of the OR should be done in the absence of any epidemiologic investigations in that area 3. Routine culturing should not be done because it is too expensive 4. Routine environmental culturing should not be considered unless an epidemiologic investigation is being conducted

4 microbiological environmental testing is generally not recommended as it it costly and can require special procedures. also there is no standard for comparison. may generate inconclusive data that could result in unnecessary procedures or treatment. rationale for environmental testing should be carefully planned and limited to epidemiological investigations. routine sampling should be limited to: 1) biologic monitoring of sterilization processes; 2) monthly cultures and endotoxin testing of water and diasylate in hemodialysis units; and 3) short-term eval of the impact of IP measures or changes in IP protocols

An IP subscribes to several peer-reviewed journals. As she reviews published articles describing research findings, what question should she ask before incorporating the conclusions/findings of the article into the infection prevention program: a. Was the appropriate study design used and are the conclusions reasonable? b. Is the author well known and well published? c. Does the article state how to contact the author(s) with questions? d. Are the findings described in tables or graphs and easily understandable?

A

If chance is a likely explanation for the difference between a sample statistic and the corresponding null hypothesis population value, then: a. The difference is not statistically significant b. The sample results are not compatible with the null hypothesis c. The difference is statistically significant d. The null hypothesis can be rejected

A

A 40 year old female is admitted with a 3-day history of diarrhea and fever. She is placed in contact isolation for a suspected clostridioides difficile infection (CDI). the patient also reports that she has had increasing abdominal pain for the past year, and inflammatory bowel disease (IBD) is in the list of potential diagnoses. Which of the following statements is most accurate regarding the relationship between IBD and CDI? a. IBD is associated with increased morbidity and mortality associated with CDI b. most patients with IBD acquire CDI in inpatient settings c. CDI generally develops more slowly after hospital admission among patients with IBD compared with patients without IBD d. IBD does not affect the risk of CDI from antibiotic exposure

A As CDI has become more common, CDI in individuals with IBD has become a focus of increased attention. IBD has been identified as an independent risk factor for C difficile colonization and disease; patients with IBD have increased severity of illness and higher death rates from CDI

Which statistical test is used when the data are small in numbers? a. Fisher's exact b. t test c. Chi-square d. z test

A Fisher's exact test is a statistical significance test used in the analysis of contingency tables. although in practice it is employed when sample sizes are small, it is valid for all sample sizes

Which immune marker represents past exposure to disease? a. IgG b. IgE c. IgM d. IgA

A IgG is the major circulating and extravascular (interstitial) antibody. IgG is the late-occurring immunoglobulin in an immune response and is the longest lived. IgG represents past exposure to disease

The IP monitors all patients who have coronary artery bypass graft surgery for infections and pneumonia. The probability or likelihood of an event occurring is the: a. risk b. attack rate c. host factor d. incidence

A In epi, risk is defined as the probability that an event will occur (e.g., that an individual will become ill or die within a stated period of time or age)

A patient is admitted for a skin infection after swimming in the ocean. Which of the following organisms is the most likely cause? a. Mycobacterium marinum b. Mycobacterium avium c. Mycobacterium leprae d. Mycobacterium tuberculosis

A M marinum causes cutaneous lesions after exposure to swimming pools, fish tanks, or other water sources. organisms may enter through previously unappreciated superficial nicks and abrasions. lesions first appear as papules that later ulcerate. because special culture conditions must be used to isolate the organism, the microbio lab should be alerted if this is a diagnostic consideration m leprae causes leprosy. m tuberculosis is the most common cause of of TB in US. aerobic, poorly staining, acid-fact

Which of the following is indicative of superficial SSI? a. pain at the incision site 10 days after a breast reduction procedures; drainage is culture-positive for methicillin-susceptible staphylococcus aureus (MSSA) b. stitch abscess that is cultured 14 days after surgery and is positive for enterococcus faecalis c. purulent drainage from an episiotomy that occurs within 5 days of delivery d. burn would that cultures positive for Acinetobacter baumannii 10 days after debridement procedure

A SSI continues to be a major source of morbidity, economic cost, and even death in surgical patients. to mee the criteria for a superficial SSI, the infection must occur within 30 days after the operation and involve only skin to subcutaneous tissue. in addition, one of the following must be met: -purulent drainage from the superficial incision -organism(s) isolated from an aseptically obtained culture of fluid or tissue from the superficial incision -and patient has at least one of the following: pain or tenderness, localized swelling, erythema, or heat -diagnosis or superficial incisional SSI by the surgeon or attending physician or other designee

An OR team was using a unique surgical instrument on a patient and the instrument was accidentally dropped on the floor. The team needs to reprocess it as quickly as possible to finish the surgery and would like to use Immediate-Use Steam Sterilization (IUSS). Which of the following is the best recommendation for this? a. The team must clean and inspect the instrument before proceeding with IUSS b. The team can proceed with IUSS without prior cleaning and inspection of the instrument since it will be used on the same patient c. There is no need to use IUSS as the team can simply soak the instrument in ten percent bleach for 10 minutes d. There is no situation in which IUSS is acceptable

A a sterilization method (formerly called "flash" sterilization) that involves the shortest possible time between a sterilized item's removal from the sterilizer and its aseptic transfer to the sterile field. a sterilized item intended for immediate use is not stored for future use nor held from on case to another. eliminates packaging and drying time. riskier than sterilization in SP if people skip or miss steps in a rush, can result in contamination. only appropriate for urgent clinical situations for this reason and should be minimized. procedures for cleaning soiled instruments and rigid containers should follow manufacturers IFU. cleaning is most critical step.

Respiratory hygiene and cough etiquette includes all of the following except: a) covering the mouth and nose with the hands when coughing and sneezing b) offering a surgical mask to a coughing patient c) discarding used masks and tissues appropriately and performing hand hygiene d) posting signs in public areas in languages appropriate to the population served and education health care staff, patients, and visitors

A according to the CDC, respiratory hygiene and cough etiquette strategies are used to prevent the transmission of all respiratory infections in health care settings. respiratory hygiene and cough etiquette include covering the mouth and nose with a tissue during coughing and sneezing or offering a surgical mask to the coughing patient; discarding the mask or tissue appropriately and performing hand hygiene; posting signs in public areas in languages appropriate to the population served; and educating health care staff, patients, and visitors

Which of the following terms refers to patient harm that is the result of treatment by the health care system rather than from the health condition of the patient? a. adverse event b. dire consequence c. unanticipated event d. sentinel event

A an adverse event is an unintended consequence of health care or services that results in a negative patient outcome (eg, infection or physical or psychological injury). Incidents such as patient falls or improper administration of medications are also considered adverse events even if there is no permanent effect on the patient

You are working with the antimicrobial stewardship program in your facility to educate providers on appropriate use of antimicrobials. Which of the following would be an accurate recommendation from your group? a. Metronidazole is an effective antimicrobial to use for MRSA skin infections b. Consistent use of broad-spectrum antimicrobials regardless of susceptibility results will help reduce antimicrobial resistance in the facility c. It is not always necessary to maintain antimicrobial concentration in the body above the level of the minimum inhibitory concentration of the pathogen being targeted d. Cefepime is only useful for gram-negative bacterial infections

A cefepime can be used for gram neg and pos bacteria

Which of the following does not describe indirect contact transmission? a) mites from a scabies-infested patient b) equipment that is not cleaned, disinfected, or sterilized adequately between patients c) food and water supplies that are not prepared and maintained according to sanitation standards d) inadequate hand hygiene performed by a care provider

A contact transmission is the most common mode of transmission and is divided into two subgroups: direct and indirect contact. mites from a scabies-infested patient are an example of direct contact transmission indirect transmission involves the transfer of an infectious agent through a contaminated intermediate object or person. Hands of health care personnel may transmit pathogens after touching an infected or colonized body site on one patient or a contaminated inanimate object. Transmission may occur if hand hygiene is not performed; if equipment is inadequately cleaned, disinfected, or sterilized; or if there is exposure to contaminated food and water that were not prepared and maintained according to sanitation standards

An IP is reviewing the cerebrospinal fluid (CSF) results from a patient admitted the previous night. The CSH is cloudy and has an elevated WBC, markedly increased neutrophils, low glucose level, and elevated protein concentration. What type of meningitis should she suspect? a. bacterial b. viral c. fungal d. aseptic

A diagnosis of bacterial meningitis rests on examination of the CSF. the CSF appearance is typically cloudy, depending on the presence of significant concentrations of WBCs, RBCs, bacteria, and/or protein. in untreated bacterial meningitis, the WBC count is elevated. bacterial meningitis usually leads to a neutrophil predominance in CSF, typically between 8- and 95%. the CSF glucose concentration is <40 mg/dL in approx 50 to 60% of patients. the CSF protein concentration is elevated in virtually all patients with bacterial meningitis.

The safe temperature range for cold food storage is: a. 41°F/5°C or lower b. 42°F to 50°F/5.6°C to 10°C c. 50°F to 55°F/10°C to 12.8°C d. Less than 60°F/15.6°C

A for cold foods for serving at 41F or lower. the temperature danger zone is 41F to 135F, which is the temperature range for rapid multiplication of virtually all bacteria associated with foodborne disease

All of the following organisms can penetrate the intact epithelium of the conjunctive or cornea except: a. staphylococcus aureus b. streptococcus pneumoniae c. neisseria meningitidis d. neisseria gonorrhoeae

A health care-related ocular infections are rare but may lead to catastrophic consequences such as compromised vision and/or blindness. only a few organisms can penetrate the intact epithelium of the conjunctiva or cornea. among these are neisseria gonorrhoeae, neisseria meningitidis, streptococcus pneumoniae, listeria monocytogenes, and corynebacterium diptheriae. For all others, a break in the protective epithelial barrier or mucous membranes must occur.

Which of the following recommendations related to disinfection and sterilization in healthcare facilities is a CDC category 1A recommendation? 1) "Before use on each patient, sterilize critical medical and surgical devices and instruments that enter normally sterile tissue or the vascular system or through which a sterile body fluid flows" 2) "Meticulously clean patient-care items with water and detergent, or with water and enzymatic cleaners before high-level disinfection or sterilization procedures" 3) "In hospitals, perform most cleaning, disinfection, and sterilization of patient-care devices in a central processing department in order to more easily control quality" 4) "Perform low-level disinfection for noncritical patient-care surfaces (e.g., bedrails, over-the-bed table) and equipment (e.g., blood pressure cuff) that touch intact skin" a. 1 b. 1, 3 c. 1, 2, 4 d. 1, 2, 3, 4

A https://www.cdc.gov/infectioncontrol/pdf/guidelines/disinfection-guidelines-H.pdf page 83-84

The measure of central tendency most affected by outliers is: a. mean b. median c. mode d. range

A measures of central tendency describe how observations cluster around a middle value and locate only the center of a distribution measure. The methods include mean, median, and mode. The most commonly used parameter is the arithmetic mean (average). The mean of a data set is inaccurate if there are extreme values (outliers) in a data set. Most statistical tests use the mean because it is more amendable to mathematical manipulation than the median or the mode. However, because the mean includes the value of each observation, it is the measurement most affected by outliers (unusually high or low values), especially when the number of observations is small. as the sample size gets very large, outliers are less important.

The following blood culture result should be considered a potential contaminant: a. a positive result of coagulase-negative staphylococci from two sets, 2 days apart, without symptoms b. a positive result of S. aureus from one bottle in a patient with a temperature of 38.6C c. a positive result of E. coli from one bottle in an afebrile patient with diarrhea d. a positive result of candida albicans in a fungal blood culture in a patient with a UTI

A more than 30 identified species/subspecies of coagulase-negative staphylococci. colonize mucosa and skin. although CoNS are the most common organisms isolated in blood cultures, a large proportion of these isolates represent events of contamination rather than true bacteremia. several indicators can be used to identify true CoNS bacteremia, including occurrence of multiple positive blood culture sets, growth in both aerobic and anaerobic bottles, detection of positive blood cultures less than 48 hours after cultures are obtained, and presence of foreign bodies in the patient, which predispose to CoNS infection e coli causes diarrhea but doesn't always have fever and usually isnt high if there is. s. aureus can fever. C. albicans fourth most common cause of catheter-associated UTI's. unsure about bottle as mode of transmission?

The Emergency Department reports three cases of cramping, abdominal pain, and diarrhea within a 24-hour period. All persons are from the same community, and onset of symptoms was within 12 to 36 hours of a picnic they all attended. The IP suspects which of the following foodborne illnesses: a. Salmonella b. Hepatitis A c. Staphylococcus aureus d. Clostridioides perfringens

A most persons infected with Salmonella develop diarrhea, fever, and abdominal cramps 12 to 72 hours after infection. The illness usually lasts 4 to 7 days, and most persons recover without treatment. However, in some persons, the diarrhea may be so severe that that patient needs to be hospitalized. In these patients, the Salmonella infection may spread from the intestines to the bloodstream and to other body sites, possibly resulting in death if the person is not treated promptly with antibiotics. Infants, elderly persons, and those with impaired immune systems are more likely to have a severe illness from Salmonella

Prior to opening a sterile package, the end user should inspect the package for: 1) Tears 2) Moisture 3) Date of manufacture 4) The name of the person who packaged the kit a. 1, 2 b. 2, 4 c. 3, 4 d. 1, 3

A other aseptic practices involve using single-use devices and equipment or reusable devices and equipment that have been properly processed and packaged. personnel should maintain the sterile packaging and/or container integrity to ensure an intact seal and confirm that sterilization indicators with expiration date (if provided) are verified. before use, sterile packages should always be inspected for signs of contamination such as moisture, tears, or discoloration in addition to the expiration date. items including solutions should be aseptically transferred to sterile fields. when used for sterile procedures, disinfectant pads or swabs including those in sterile kits or trays, should be checked to verify that the label on their package indicates they are sterile--otherwise they are considered nonsterile.

A pediatric patient has been diagnosed with pediculosis. What is the most appropriate follow-up to prevent it from spreading to other patients or health care professionals? a. Place the patient on Contact Precautions until 24 hours after appropriate treatment has been initiated b. Require all visitors and HCP who enter the room to wear a disposable scrub cap for any patient contact c. Use an insecticidal spray in the room after the patent is discharged d. Prophylactically treat all family members and anyone with close physical contact with the patient

A pediculosis is an infestation with the human head-and-body louse

An urban community is experiencing an outbreak of Bordetella pertussis. Several employees have contacted the IP at their health care facility for information on the tetanus, diphtheria, and pertussis (Tdap) vaccine. They question the need for the vaccine because they received it as a child. The IP should inform them that the Advisory Committee for Immunization Practices (ACIP) recommends that: a. all adults aged 19 and older should receive at least one dose of Tdap b. if the employee is pregnant, she should not receive the vaccine c. all individuals must receive the vaccine every 10 years d. individuals who have had the disease do not need to receive the vaccine

A pertussis (whooping cough) is a highly communicable, acute, infectious respiratory disease caused by Bordetella pertussis. ACIP recommends a single Tdap dose for persons aged 11 to 18 years who have completed the recommended childhood diphtheria and tetanus toxoids and pertussis/diphtheria and tetanus toxoids and acellular pertussis (DTP/DTaP) vaccination series and for adults aged 19 to 64 years

The paroxysmal stage of pertussis usually lasts: a. 1 to 6 weeks b. 11 to 15 weeks c. 15 to 20 weeks d. 6 to 21 weeks

A pertussis is highly communicable, acute, infectious respiratory disease caused by bordetella pertussis. the onset of pertussis, known as the catarrhal stage, begins with coryza (runny nose), sneezing, low-grade fever, and a mild, occasional cough that gradually becomes more sever. The cough that began during the catarrhal stage progresses steadily, becoming paroxysmal (numerous rapid coughs). it is during this second paroxysmal stage that the diagnosis of pertussis is usually suspected. the classic symptoms of pertussis include whoop, vomiting, apnea, and cyanosis immediately after a paroxysm of coughing. infants younger than 6 months may have an atypical presentation with a short catarrhal stage, gagging and gasping

During the annual TST, an employee's test result was read as 10mm induration. The employee's last TST was negative. This initial result indicates a: a. positive test b. false positive c. negative test d. false negative

A positive test is determined by the number of millimeters of induration (not erythema) caused by the reaction to the tuberculin. Different cut points are used depending on the likelihood of the individual having a TB infection and the likelihood that that infection, if present, will progress to active TB. A 10-mm reaction in a health care worker is considered a positive test. All health care personnel with positive baseline TST results should be referred for medical and diagnostic evaluation; additional skin testing is not needed

Sensitivity may be defined as: a. the ability of a test to detect true positives (persons with the disease) when applied to a population with the disease b. the ability of a test to detect the true negatives (persons without the disease) when applied to a population without the disease c. the ability of a test to detect true positives (persons with disease) when applied to a population without the disease d. the percentage of persons with true positive results when the test is applied to persons without the disease

A sensitivity: the ability of a test or other surveillance method to identify true cases. if someone has the outcome, what is the likelihood the test will be positive? specificity: the ability of a test or other surveillance method to exclude persons who are not cases. if someone does not have the outcome, what is the likelihood the test will be negative? positive predictive value PPV: if the test result is positive, what is the likelihood that the person truly has the outcome? negative predictive value NPV: if the test result if negative, what is the likelihood that the person truly does not have the outcome?

The Director of Infection Prevention and Control has been asked to participate in the organization's strategic planning. Which of the following might be a strategic goal for the Infection Prevention and Control program? a. implement an electronic surveillance system in the next 3 years b. fill the vacant IP position in the department within 45 days c. participate more actively in the organization's value analysis committee d. share key HAI reports with senior managers every month

A strategic planning is an organization's process of defining its strategy, or direction, and making decisions on allocating its resources to pursue this strategy. the process includes setting goals, determining actions to achieve the goals, and mobilizing resources to execute the actions. strategic goals are planned objectives that a department or organization strives to achieve. Answer A is an example of a clear measurable goals that is focused on the future and provides direction for the department

Targeted surveillance focuses on: a. tracking high-risk, high-volume procedures and potentially preventable health-care associated infections (HAIs) b. providing whole-house infection rates c. tracking infections that are publicly reported d. using the electronic surveillance systems to identify infections

A targeted surveillance is one method used for HAI surveillance. targeted surveillance focuses on particular units, invasive procedures, infections related to medical devices, and organisms of epidemiological significance

Failure mode and effects analysis (FMEA) is used to examine adverse events and identify what went wrong and what might prevent it from happening again. Which statement best describes the mode element of FMEA? a. the way the operating or using a system or process, or a way or manner in which a thing is done b. the results or consequences of an action c. the detailed examination of the elements or structure of something-perhaps a process, substance, or situation d. lack of success, nonperformance, nonoccurrence, or breaking down or ceasing to function

A the FMEA tool is a proactive, preventive approach to identify potential failures and opportunities for error. the mode is described as the way of operating or using a system or process, or a way or manner in which a thing is done. A mode is the way or manner in which something, such as a failure, can happen. Combining the words "failure" and "mode", a failure mode is the manner by which something can fail. a failure mode generally describes the way the failure occurs and its impact on a process. any step in a process can fail, and each failure may have many failure mode

Which of the following milestones indicates that the IP has achieved proficient status according to the APIC competency model? a. after successfully completing his or her certification in infection control b. upon obtaining a graduate degree in a health care-related field c. when continuously employed as an IP for more than 2 years d. after 10 years of experience as the manager of an infection prevention and control program

A the certification in infection control credential identifies health care professionals who have shown mastery in knowledge in infection prevention and control by sitting for an passing the certification exam. according the APICs competency model, the proficient Ip has earned an undergraduate degree and is often pursuing post-baccalaureate education. the proficient IP may have management or supervisory responsibility. this IP has earned certification and may serve as a mentor for those pursuing the credential. proficient IPs have a diverse skill set, demonstrate critical thinking, and function successfully in team-based, collaborative situations. They have further developed and are refining their leadership skills and are effectively managing their IPC program. proficient IPs are highly skills and professionally confident in their roles as preventionists and patient safety advocates

Measures that can be practiced for prevention of aspiration include all of the following except: a. antibiotic prophylaxis b. oropharyngeal cleaning and decontamination with an aseptic agent (e.g. chlorhexidine) c. orotracheal intubation, unless contraindicated, rather than nasotracheal intubation d. the head of the bed elevated at an angle of 30-45 degrees

A the following precautions should be practiced for prevention of aspiration: -use of noninvasive ventilation, when possible, to reduce the need for and duration of endotracheal intubation. this refers to all modalities that assist ventilation without the use of an endotracheal tube -perform orotracheal intubation unless contraindicated. nasotracheal intubation has been associated with higher incidence of nosocomial sinusitis, making the patient more prone to development of pneumonia through aspiration of infected secretions -the head of the bed should be elevated at an angle of 30-45 degrees -oropharyngeal cleaning and decontamination should be performed with an aseptic agent (eg chlorhexidine) -stress ulcer prophylaxis may be provided with proton-pump inhibitors, histamine-2 receptor antagonist, or sucralfate

Which one of the following statements is true regarding bacterial spores? a. They are resistant to antibiotics b. They allow the bacteria to multiply in adverse conditions c. They are usually formed by gram-negative bacteria d. They can be identified with gram stain

A the function of a spore is to permit the cell to survive unfavorable conditions such as extremes of temperature or moisture. Spores becomes dormant at these times; they do not multiply in adverse conditions. Instead, the spore protects the bacteria until favorable conditions occur, at which time the bacteria can begin to multiply. Spores are usually formed by gram-positive bacteria

Gram stains classify an organism as gram-positive or gram-negative. The determinant factors for gram stains are cell wall components of: a. Peptidoglycans b. Lipids c. Polysaccharides d. Mycolic acids

A the gram stain is the most important and universally used staining technique in the lab. it is used to distinguish between gram-positive and gram-negative bacteria, which have distinct and consistent differences in their cell walls. Gram-positive bacteria have thick peptidoglycan cell wall that does not allow the crystal violet/iodine complex to be removed during the alcohol wash. Under the microscope, Gram-positive organisms appear dark violet, purple, or blue. Gram-negative bacteria contain a lipopolysaccharide layer as part of their cell wall. The alcohol wash disrupts this layer, and the crystal violet/iodine complex is rinsed out of the cell wall. As a result, gram-negative cells are colorless until counterstained with safranin. Under the microscope, gram-negative organisms appear pink or red

If the index of kurtosis is -1.99, then the curve is: a. relatively flat b. negatively skewed c. more peaked d. a typical bell-shaped curve or normal distribution

A two terms are used to describe the shape of a frequency distribution: "skewness" and "kurtosis". Kurtosis refers to how flat or peaked a curve is: -mesokurtic is a typical bell-shaped curve or normal distribution (0) -leptokurtic is more peaked curve (+) -platykurtic is the flatter curve (-) statistical packages calculate kurtosis. a value of 0 indicates mesokurtosis, positive numbers indicate leptokurtosis, and negative numbers indicate platykurtosis

What is the probability of committing a Type I error if the p value is 0.10? a. 1 in 10 b. 1 in 100 c. 1 in 5 d. 1 in 20

A type 1 error means rejecting the null hypothesis when it's actually true. concluding results are statistically significant when, in reality, they came about purely by chance or because of unrelated factors. risk of of committing this error is significance level (alpha). significance is usually set at 0.05 meaning that your results only have a 5% chance of occurring, or less, if the null hypothesis is actually true. type 2 error means not rejecting the null hypothesis when it's actually false. failing to conclude there was an effect when there actually was.

Which of the following is not likely to contaminate total parenteral nutrition? a. Mycobacterium fortuitum b. Candida albicans c. Pseudomonas aeruginosa d. Staphylococcus epidermidis

A with strict adherence to aseptic compounding technique, contamination of total parental nutrition (TPN) solution is rarely the cause of sepsis. However, TPN can foster microbial growth. Organisms that have been reported to proliferate in TPN include fungi (candida albicans or malassezia furfur); gram-positive bacteria (coagulase-negative staphylococcus, staphylococcus saprophyticus, or staphylococcus epidermidis); and gram-negative bacteria (escherichia coli or pseudomonas aeruginosa). Candida is one of the most frequently reported organisms

The most important feature of nonparametric tests is that they: a. make no assumption about variance in the populations b. can only be used with ordinal levels of measurements c. require a normal distribution d. require equal population variances

A nonparametric data make no assumption about the distribution of the population values and can be used with discrete data (eg infection, no infection), nominal and ordinal data, and interval data. The main advantage of nonparametric methods is that the assumptions of normality are not required

A 14-year-old boy from rural Maryland was seen in the emergency room with fever, fatigue, chills, headache, and a large annular lesion on his left thigh, which the patient described as burning and itching. What is the most probably vector of this child's illness? a. Tick b. Mosquito c. Flea d. Louse

A this child's symptoms are consistent with Lyme disease. Typical symptoms include fever, headache, fatigue, and a characteristics skin rash called the erythema migrans. If left untreated, infection can spread to joints, the heart, and the nervous system. Lyme disease is diagnosed based on symptoms, physical findings (eg, rash), and the possibility of exposure to infected ticks; lab testing is helpful if used correctly and performed with validated methods. The black-legged tick, commonly known as deer tick, can transmit the organisms responsible for anaplasmosis, babesiosis, and lyme disease. This tick is widely distributed in the northeastern United States

The incubation period for pertussis in immunocompetent persons is usually: a. 7 to 10 days b. 3 to 5 days c. 1 to 2 days d. 2 to 4 days

A 7 to 10 days pertussis (whooping cough) is highly communicable, acute, infectious respiratory disease caused by Bordetella pertussis. the incubation period of pertussis in immunocompetent patients is usually 7 to 10 days, with a range of 6 to 21 days. in rare cases, the incubation period may be as long as 42 days.

The optimal time to collect a sputum specimen for acid-fast bacilli (AFB) testing to rule out TB would be: a. first thing in the morning b. after a respiratory treatment c. prior to the patient going to bed d. prior to a respiratory treatment

A first thing in the morning because pulmonary disease is the most common form of TB, patients with suspected TB should have a chest radiograph. If the radiograph is abnormal or the patient has respiratory complaints, sputum specimens should be collected for acid-fast bacilli (AFB) staining, culture, or other direct tests. The first sputum may be obtained on admission, but subsequent cultures are usually obtained in the morning on consecutive days. A total of three specimens collected on separate days is adequate, and once one specimen is AFB positive, subsequent samples are not needed. Specimens should be delivered promptly to the laboratory for processing

An employee has experienced an accidental needlestick injury while providing care to a patient. All of the following lab tests would be appropriate for the source patient except: a. Human immunodeficiency virus (HIV) b. Hepatitis B antibody c. Hepatitis B surface antigen d. Hepatitis C Antibody (Anti-HCV)

B anybody who has been vaccinated for hepatitis B will test positive for the antibody. test for antigen instead.

Which of the following is an example of the criterion of "Strength of the Association" from Hill's criteria for causation? a. In a study of the association between antibiotic exposure and development of C. difficile infection, the odds ratio was 2:3 b. In a study of the association between antibiotic exposure and development of C. difficile infection, the authors' conclusions are consistent with those of three other studies c. In a study of the association between antibiotic exposure and development of C. difficile infection, antibiotic therapy began an average of 3 weeks before C. difficile infection developed d. In a study of the association between antibiotic exposure and development of C. difficile infection, prolonged antibiotic therapy was a greater risk factor for C. difficile infection that short-term antibiotic therapy

A. strength of association: incidence of disease should be higher in those who are exposed to the factor under consideration than in those who are not exposed; that is, the stronger the association between an exposure and a disease is, the more likely the the exposure is the be causal. b. shows consistency (association should be observed in numerous studies) c. shows temporality (exposure to the hypothesized causal factor must precede onset of disease) d. shows biological gradient (dose-response. increased exposure to a factor and increased likelihood of disease)

An acute care facility experiences an outbreak of Serratia marcescens bloodstream infections. After the outbreak is under control and no new cases are being reported, the IP wants to find the source of the outbreak. The most appropriate epidemiology study design to use is: a. Retrospective cohort study b. Prospective cohort study c. Case-control study d. Cross-sectional study

C

As a time-saving measure, hospital administrators in your facility have suggested that endocavitary probes do not need to undergo high-level disinfection because they are used with probe covers. How would you respond to this suggestion? a. Agree because the probe cover prevents contact with mucous membranes and non-intact skin b. Disagree because the probe covers are not 100 percent reliable c. Agree, because even without covers the probes don't contact mucous membranes and non-intact skin during use d. Disagree because switching from high-level disinfection to low-level disinfection will not save time

B

The biological indicator that was included in a steam sterilization load of non-implantable instruments has shown a positive result, as have biological indicators used in two additional follow-up tests of the sterilizer. The chemical indictor in the original load was reactive, as were the chemical indicators used in the follow-up tests. The log of all the runs indicates that the run conditions (temperature and time) were appropriate for the instrument load and there were no abnormalities in steam supply or electrical supply. Which of the following should be done next: a. The load should be released for use in the facility b. The sterilizer should be tested again with paired biological indicators from two different manufacturers c. The items from the load should be immediately recalled and reprocessed d. All items that were processed in that sterilizer since the time of the last negative biological indicator should be recalled and reprocessed.

B

Which of the following is an accurate statement regarding the antibiogram for Staphylococcus isolates shown below? 1) 50 percent of Staphylococcus aureus isolates were resistant to Vancomycin 2) 12 percent of MRSA isolates were resistant to Trimeth/Sulfa 3) Clindamycin should not be used for coagulase-negative Staphylococcus infections in this facility because 100% of isolates were resistant to it 4) There were no cases of Vancomycin-resistant Staphylococcus aureus in this facility a. 2 b. 2, 4 c. 1, 2, 4 d. 1, 2, 3, 4

B

An employee has sustained a needlestick injury from a blood-contaminated needle. The source patient was a Hepatitis B virus (HBV) positive, and the employee had completed one of the three vaccinations in the Hepatitis B series. Which of the following is the correct postexposure prophylaxis (PEP) for this patient? a. Complete the Hepatitis B vaccine series b. Complete the Hepatitis B vaccine series and provide Hepatitis B immunoglobulin c. Provide Hepatitis B immunoglobulin and begin interferon therapy d. No PEP is needed

B HBV is transmitted by percutaneous or mucosal exposure to infectious blood or body fluids. The risk of HBV seroconversion after a percutaneous injury ranges from 23 to 62 percent depending on the Hepatitis B e antigen (HBeAg) status of the source person. For exposed persons who are in the process of being vaccinated but have not completed the vaccination series, vaccination should be completed as scheduled, and Hepatitis B immunoglobulin (HBIG) should be added as indicated. Both HBIG and the Hep B vaccine should be administered as soon as possible after exposure (preferable within 24 hours)

Higher morbidity rates in chronic Hepatitis B virus (HBV) carriers are associated with co-infection of which of the following: a. Hep A b. Hep D c. Hep C d. Hep E

B Hep D, also known as "delta hepatitis", is a serious liver disease caused by infection with HDV, which is an RNA virus structurally unrelated to the Hep A, B, or C viruses. Hep D, which can be acute or chronic, is uncommon in the US. HDV is an incomplete virus that requires the helper function of HBV to replicate and only occurs among people who are infected with HBV. HDV is transmitted through percutaneous or mucosal contact with infectious blood and can be acquired either as a co-infection with HBV or as a superinfection in persons with HBV infection. There is no vaccine for Hep D, but it can be prevented by Hep B vaccination in persons who are not already HBV infected

Which of the following viruses is the causative agent in Kaposi's sarcoma? a. Herpes zoster virus b. Human herpesvirus 8 c. Epstein-Barr virus d. Human papillomavirus (HPV)

B Kaposi's sarcoma is a tumor caused by human herpesvirus 8. Kaposi's sarcoma is a systemic disease that can present with cutaneous lesions with or without internal involvement. Kaposi's sarcoma lesions are nodules or blotches that may be red, purple, brown, or black and are usually papular. They are typically found on the skin, but spread elsewhere is common, especially the mouth, GI tract, and respiratory tract. growth can range from very slow to explosively fast and is associated with significant mortality and morbidity

A patient in the ER is diagnosed with bacterial meningitis due to Neisseria meningitidis. The patient was not properly isolated, and a number of employees entered her room without wearing a mask. Which employee should receive PEP? a. The phlebotomist who drew blood on the patient b. The respiratory therapist who intubated the patient c. The radiology technician that performed the chest radiograph d. The employee from admissions that registered the patient

B PEP is advised for persons who have had intensive, unprotected contact with infected patients. Unprotected means without wearing a mask, and intensive contact would be mouth-to-mouth resuscitation, endotracheal intubation, endotracheal tube management, or close examination of the oropharynx. Prophylactic therapy should be administered immediately after the unprotected exposure. Current recommended regimens to eradicate carriage are rifampin 600mg orally every 12 hours for 2 days; a single dose of ciprofloxacin 500mg orally; or a single dose of ceftriaxone 250mg intramuscularly. Rifampin and ciprofloxacin are not recommended for pregnant women.

The lead IP has proposed using an electronic surveillance system. Senior leadership at the health care organization now wants to know what the expected return on this investment will be. What is the IPs next step? a. Describe the project cost baseline developed from previous department budgets b. provide a synopsis of the investment and direct and indirect costs, including factors such as capital expenses, depreciation, and inflation c. project the impact of the surveillance system on hospital net revenue d. calculate the amount of time needed to pay back the initial costs of the system

B ROI is a financial ratio intended to measure the benefit obtained from an investment. A high ROI means the investment gains compare favorably to investment cost. as a performance measure, ROI is used to evaluate the efficiency of an investment or to compare the efficiency of a number of different investments

The CDC recommendations for decreasing CLABSI include all of the following except: a) Educational programs b) Routine replacement of catheters c) The use of chlorhexidine for skin antisepsis d) The use of maximal sterile barrier precautions

B The CDC guidelines for the prevention of intravascular catheter-related infections do not recommend routinely replacing CVCs, PICCs, hemodialysis catheters, or pulmonary artery catheters to prevent catheter-related infections

The Safe Medical Device Act (SMDA) falls under which US federal program? a. Centers for Disease Control and Prevention (CDC) b. Food and Drug Administration (FDA) c. National Institutes of Health (NIH) d. Agency for Healthcare Research and Quality (AHRQ)

B The FDA falls within the executive branch of the US government under the Department of Health and Human Services. The FDA develops, implements, monitors, and enforces standards for the safety, effectiveness, and labeling of all drugs and biologics, including food, blood and blood products, medical and radiological devices, antimicrobial products, and chemical germicides used in conjunction with medical devices

The Infection Prevention Manager observes increasing fatigue and burnout among the infection prevention team. What should the manager do first? a. contact human resources for assistance b. gather the team to identify issues and share concerns c. initiate a corrective action plan for the group d. recommend incentives to increase job satisfaction

B although some workplace stress is normal, excessive stress can result in increased absenteeism and turnover rates, as well as decreased productivity. workplace stress can, however, be successfully reduced through organizational and worker-focused interventions. the first step involves identifying the problems and discuss the opportunities that exist for improvement and change

Antimicrobial stewardship promotes the judicious use of antimicrobials to: a. increase antimicrobial selective pressure b. ensure that the right therapy is given to the right patient with the right dose and duration c. support the development of new antimicrobials d. contain health care costs

B antimicrobial stewardship refers to coordinated interventions designed to improve and measure the appropriate use of antimicrobials by promoted the selection of the optimal antimicrobial drug regimen, dose, duration of therapy, and route of administration, Antimicrobial stewards seek to achieve optimal clinical outcomes related to antimicrobial use, minimize toxicity and other adverse events, reduce the costs of health care for infections, and limit the selection for antimicrobial-resistant strains

The epidemiological triangle includes a model of dynamic interaction, where a change in any component alters the existing equilibrium. This model is particularly useful in the study of infectious disease. How can IPs use the epidemiological triangle during outbreak investigations? a. examination of host factors such as increasing antimicrobial resistance as a result of antibiotic pressure b. examination of host factors such as changes in immunity or diagnostic and therapeutic procedures within populations served c. consideration of agent factors such as improving influenza vaccination rates for health care workers d. consideration of agent factors such as a change in cleaning agents used in the hospital environments

B The epidemiological triangle consists of three elements: host, agent, and environment. The host is the human, the environment consists of all external factors associated with the host, and the agent may be a bacteria, virus, fungi, etc. Within the model presented, increasing antimicrobial resistance represents a change in the agent (or pathogen), not a change in the host. Improving health care personnel influenza vaccination rates would be environmental factor change for patients by decreasing their exposure to flu. It is also a host factor change for health care personnel because vaccination improved their immunity (eg, increased numbers of immunocompromised patients) or changes in diagnostic and therapeutic procedures (such as new surgical procedures) would be significant findings for outbreak investigations. The IP can use the epidemiological triangle to analyze and communicate how these changes increased the risk of HAIs.

Elements of a ventilator-associated pneumonia (VAP) prevention bundle that have been suggested by the Institute for HealthCare Improvement (IHI) include all the following practices except: a. Elevation of the head of the bed b. Weekly "sedation vacations" and assessment of readiness to extubate c. Peptic ulcer disease prophylaxis d. Deep venous thrombosis prophylaxis

B VAP in a critically ill patient significantly increases the risk of mortality and, at a minimum, increases ventilator time, length of stay, and cost of care. The IHI Ventilator Bundle is a grouping of best practices that, when applied together, may result in substantially greater improvement. The key components of the IHI Ventilator Bundle are: -elevation of the head of the bed to 30-45 degrees -daily 'sedation vacation' and daily assessment of readiness to extubate -peptic ulcer disease prophylaxis -deep venous thrombosis prophylaxis

Which of the following statements is true regarding storage of vaccines? a. Vaccines should be taken out of the original packaging b. Vaccines should be stored in a labeled container or bin on the middle shelf, a few inches from the wall c. Vaccines should be packed tightly into the refrigerator d. Vaccines should be stored in the top of the refrigerator

B Vaccine storage and handling errors can reduce vaccine potency and result in inadequate immune response and protection against disease. The CDC recommends the following regarding vaccine storage: -vaccines need to be placed in the central area of the unit, away from walls, vents, and coils -avoid placing vaccines on the top shelf -there must be enough room to store the year's largest inventory without crowding -a calibrated thermometer should be placed inside each storage unit -the storage unit must be dedicated to the storage of vaccines

What is the name for the substance that prevents water-soluble elements such as antibiotics and disinfectants from reaching pathogens? a. cell wall b. biofilm c. sludge d. biocarbon

B a biofilm is any group of microorganisms in which cells stick to each other on a surface. these adherent cells are frequently embedded within a self-produced matrix of extracellular polymetric substance. Biofilm extracellular polymeric substance-which is also referred to as slime-is a polymeric conglomeration generally composed of extracellular DNA, proteins, and polysaccharides. Biofilms may form on living or nonliving surfaces and can be prevalent in natural, industrial, and hospital settings.

The Director of Infection Prevention and Control has been asked to develop a business plan to potentially expand the scope of the organization's infection prevention program. The objective in developing a business plan for leadership is to: a. provide a detailed synopsis of the impact of new services b. demonstrate whether the expanded program will be worth the investment c. summarize the infrastructure needs to support an expanded program d. analyze program costs during the past 5 years

B a business plan is a formal statement of a set of business goals, the reasons they are believed attainable, and the plan for reaching those goals. Business plans are decision-making tools and cost and revenue estimates are central to any business plan for deciding the viability of the planned venture

An appropriate indicator to monitor process compliance would be: a. Class 1 SSI rate b. Appropriate antibiotic dosage c. Central line-associated bloodstream infections (CLABSIs) in the Neonatal Intensive Care Unit (NICU) d. Infections cause by multidrug-resistance organisms

B a surveillance program should monitor a variety of outcomes, processes, and events, and some indicators should focus on personnel. a process measure focuses on a process or the steps in a process that that lead to a specific outcome. process measures are commonly used to evaluate compliance with desired care or support practices, or to monitor variation in these practices. examples of process indicators include medication errors; influenza vaccination rates in personnel, residents, or patients; hepatitis B immunity rates in personnel; and personnel compliance with IP protocols, such as standard precautions, isolation precautions, tuberculin skin testing, hand hygiene, instrument processing, sterilization quality assurance testing, environmental cleaning, communicable disease reporting, antimicrobial prescribing and administration, and installing and maintaining barriers during construction and renovation projects. an outcome measure is a measure that indicates the result of the performance (or nonperformance) of a function(s) or process(es). examples of outcome indicators that may be monitored include HAIs (eg bloodstream, urinary tract, pneumonia, surgical site, conjunctivitis, upper respiratory tract, or local intravenous site), infection or colonization with a specific organism (eg C diff, MRSA, vancomycin-resistant enterococci or other antibiotic-resistance organisms, respiratory syncytial virus, or rotavirus); decubitus ulcers; phlebitis related to peripheral intravascular therapy; pyrogenic reaction or vascular access infection in hemodialysis patients; resident or patient falls; influenza or tuberculin skin test conversions in patients, residents, or health care providers; and sharps injuries and bloody/body fluid exposures in health care providers

When a study is completed, a report should be written to give the results and evaluation of the study. A good way to display data is in charts or tables. A table is used to illustrate data: a. using only one coordinate b. arranged in rows and columns c. using a system of coordinates d. showing multiple complex factors at one time

B a table is an organized set of data elements (values) that uses a model of vertical columns (which are identified by their name) and horizontal rows. The cell is the unit where a row and column intersect. A table has a specified number of columns but can have any number of rows. each row is identified by the values appearing in a particular column subset that has been identified as a unique key index

An employee is exposed to a known HIV-positive patient's blood via needlestick after giving an intramuscular injection. The patient has a known high viral load. After the employee has thoroughly washed the exposed area with soap and water, what is the next step that should be taken following this exposure? a. The employee needs to be counseled about using safer sex practices and to avoid pregnancy, breast-feeing, and blood and organ donation for 3 months after exposure b. The employee should be treated as soon as possible with expanded multidrug PEP c. The employee should have baseline testing for HIV, Hep B antigen, and Hep B antibody d. The employee should be counseled by a clinician knowledgeable about HIV transmission risks

B because the patient is known to have a high HIV viral load, the exposure should be considered an increased risk for transmission. According to the CDC, most HIV exposures warrant a two-drug regimen using two nucleoside reverse transcriptase inhibitors (NRTIs) or on NRTI and one nucleotide reverse transcriptase inhibitor. The CDC recommends that the addition of a third (or even fourth) drug should be considered for exposures that pose an increased risk for transmission or that involve a source in whom antiretroviral drug resistance is likely.

The manager notices that a novice IP has misapplied the CDC definitions at least five times when conducting catheter-associated UTI surveillance recently. How should the manager respond? a. schedule the IP for additional training and competency-based testing b. speak with the IP to obtain additional information about the situation c. use the organization's disciplinary action process to correct poor performance d. refer the IP to the employee assistance program for personal counseling

B discrepancies between current and desired job performance as well as gaps between existing and desired competencies and skills should be investigated and clarified before identifying a corrective action

OSHA mandates that which of the following vaccines be provided at no cost to health care providers and others at risk for blood and body fluid exposure. a. Hep A b. Hep B c. BCG d. Meningococcal

B exposure to bloodborne pathogens poses a serious risk to health care personnel. Avoiding occupational blood exposures through adherence to Standard Precautions and other safe work practices is essential. The most effective means to prevent transmission of bloodborne pathogens in health care settings include Hep B vaccination, the use of appropriate barriers to prevent blood and body fluid contact, and preventing percutaneous injuries by eliminating unnecessary needle use, implementing devices with safety features, using safe work practices when handling needles and other sharp devices, and safely disposing of sharps and blood-contaminated materials. OSHAs blood borne pathogen standard mandates provision of Hepatitis B vaccine at no cost to all health care personnel and others are occupational risk for blood exposure

All of the following methods for measuring hand hygiene adherence are acceptable except: a. using electronic systems that allow continuous monitoring over time and automatic data download and analysis b. monitoring the volume of gloves used per 1000 patient days c. monitoring adherence to artificial fingernail policies d. periodically conducting an observational study to determine the rate of adherence (number of hand hygiene episodes performed/number of hand hygiene opportunities) by ward or service

B hand hygiene is a critical component of patient and employee safety. evaluation and repeated monitoring of hand hygiene practices, as well as health care personnel and senior managers' knowledge and perception of the problem of HAI and the importance of hand hygiene at the health care facility, is a vital component of any successful hand hygiene campaign. Unobtrusive direct observation of hand hygiene practices by a trained observer is considered the gold standard for evaluating compliance. Electronic systems for the automatic monitoring of hand hygiene compliance are now available and can significantly facilitate data collection. consumption of hand hygiene products such as soap and alcohol-based hand rub is another useful indicator. Adherence to artificial fingernail policies may also be monitored. Health care personnel should receive feedback about defective practices as well as improvement strategies

Which of the following scenarios would be most appropriate for immediate-use sterilization: a. The vendor brings the instrument for the procedure the morning of the surgery, which does not allow for the full sterilization process b. The instrument used for the procedure is dropped on the floor of the operating room and another instrument is not available c. The turnaround time between procedures does not allow enough time for the full sterilization process d. The OR does not have the needed instruments to meet the demand of surgeries, so the instruments are flashed between procedures

B immediate-use sterilization (IUSS): a steam sterilization cycle that does not use full sterilization exposure cycles and dry times; previously referred to as "flash sterilization" IUSS should be used only when there is an urgent need for the items. surgical implants and devices intended for single-use should not be sterilized by IUSS. IUSS should be done in a space that allows for direct delivery of sterilized items to the point of use. to ensure that IUSS is used rarely and only in emergency situations, vendor trays should be available to the facility at least 48 hours before surgery to allow for proper inspection of contents, cleaning, and sterilization before surgery.

Each year in the United States, what percentage of hospitalized patients develop HAIs? a. less than 2 percent b. 4 percent c. 10 percent d. 20 percent

B in march 2014, the CDC released new data on health care-associated infection rates in US hospitals. According to the Multistate Point-Prevalence Survey of Health Care-Associated Infections, 1 in 25 patients (722000 infections) in the US acquire HAIs each year, and approximately 75000 patients who have an HAI will die during hospitalizations. The report notes that pneumonia is now the most common HAI in the US, accounting for 22 percent of infections. The second most common infections are surgical site (22 percent), followed by gastrointestinal (17 percent), urinary tract (13 percent), and bloodstream infections (10 percent). The report also notes that the top organisms leading to HAIs are Clostridioides difficile (12 percent), Staphylococcus (11 percent), Klebsiella (10 percent), E coli (9 percent), Enterococcus (9 percent), and Pseudomonas (7 percent)

When are IgM antibodies to Hepatitis A virus (HAV) detectable in the blood? a. Within 24 hours of exposure b. Within 3 weeks of exposure c. 30 days after exposure d. 8 to 12 weeks after exposure

B within 3 weeks of exposure clinical features of acute hepatitis are not specific for HAV infection so serological diagnosis is necessary. IgM antibodies to HAV, which are used to diagnose, are detectable within 3 weeks of exposure and are present at the onset of jaundice. titer declines over 4 to 6 weeks and antibodies are usually not detectable after 6 to 12 months. IgG is also detectable at onset of jaundice and remains positive lifelong, indicating immunity to hAV

An RN is caring for a patient who presented to the Emergency Department with symptoms consistent with influenza. When caring for his patient, she should use which of the following types of precautions? a. Standard precautions b. Standard and droplet precautions c. Airborne precautions d. Droplet precautions if influenza is confirmed

B influenza viruses are spread from person to person primarily through large-particle respiratory droplet transmission. Transmission via large-particle droplets requires close contact between source and recipient persons, because droplets do not remain suspended in the air and generally travel only a short distance (3 feet or less) through the air. Droplet precautions are intended to prevent transmission of pathogens spread through close respiratory or mucous membrane contact with respiratory secretions. Standard precautions to all patients, regardless of suspected or confirmed infection status

Which of the following statements about influenza false? a. influenza is primarily spread between individuals via respiratory secretions (droplets) b. viral shedding starts 48-72 hours after infection and typically 48 hours before the onset of symptoms c. viral shedding normally persists for less than 5 days but can be longer in children and immunocompromised persons d. the typical influenza symptomology is not always predictive of influenza in elderly or immunocompromised persons

B influenza viruses are spread from person to person primarily through large-particle respiratory droplet transmission. transmission via large-particle droplets requires close contact between source and recipient, because droplets do not remain suspended in the air and generally travel a short distance (~3 ft) through air. contact with respiratory droplet contaminated surfaces is another possible source of transmission. typical incubation period for influenza is 1 to 4 days (~2 avg). Adults shed influenza from the day before symptoms begin through 5-10 days after illness onset. However, the amount of virus shed, and presumably infectivity, usually decreases rapidly by 3-5 days after onset. young children also might shed virus several days before illness onset, and children can be infectious for 10 or more days after onset of symptoms. severely immunocompromised persons can shed virus for weeks or months.

The spirochete Borrelia burgdorferi is the agent responsible for: a. Legionnaires' disease b. Lyme disease c. Aseptic meningitis d. Syphilis

B lyme disease is caused by the bacterium Borrelia burgdorferi and is transmitted to humans through the bite of infected black-legged ticks. typical symptoms include fever, headache, fatigue, and a characteristics skin rash called erythema migrans. Lyme disease is diagnosed based on symptoms, physical findings (e.g. rash), and the possibility of exposure to infected ticks; laboratory testing is helpful if used correctly and performed with validated methods. Most cases of Lyme disease can be treated successfully with a few weeks of antibiotics. Steps to prevent Lyme disease include insect repellant, removing ticks promptly, applying pesticides, and reducing tick habitat.

The antibiogram is usually prepared by: a. infection prevention and control department b. laboratory c. pharmacy d. information technology department

B many hospital laboratories routinely perform antimicrobial susceptibility testing on bacterial pathogens. Cumulative susceptibility testing results are often organized into a summary table, or antibiogram, which may be used by clinicians, pharmacists, infection control personnel, and microbiologists as a reference guide to community or hospital-specific resistance patterns. antibiograms lend information that can be used to raise awareness of resistance problems, support the use of optimal empiric therapy, and identify opportunities to reduce inappropriate antibiotic usage and to ascertain success of such efforts. antibiograms are generally prepared by the laboratory according to the clinical laboratory standards institute guidelines

A measure of dispersion that reflects the variability in values around the mean is called the: a. variance b. standard deviation c. range d. bell curve

B measures of dispersion describe the degree of variation or dispersion of values in a population or in a sample. measures of dispersion are a type of descriptive statistic. measures of dispersion include the range, deviation, standard deviation, and variance. standard deviation is a measure of dispersion of the raw scores that reflects the variability in values around the mean. the standard deviation indicates how small the variability is (i.e. the spread) among observations. if the variability is small, all the values are close to the mean. if it is large, the values are not close to the mean.

Routine microbiologic sampling is indicated for which of the following? a. respiratory therapy equipment b. dialysis fluid c. sterile disposable equipment d. operating room surfaces

B microbiological environmental testing is not generally recommended. environmental culturing can be costly and may require special lab procedures. additionally, in most cases no standards for comparison exist. because of the lack of standards, environmental testing may generate inconclusive data that could result in the implementation of unnecessary procedures or treatment. routine microbiologic sampling for quality assurance purposes should be limited to 1) biologic monitoring of sterilization processes, 2) monthly cultures and endotoxin testing of water and dialysate in hemodialysis units, and 3) short-term evaluation of the impact of infection prevention measure or changes in infection prevention methods

Noninfectious postoperative endophthalmitis is most often associated with: a. Wearing contact lens b. Toxic anterior segment syndrome (TASS) c. Conjunctivitis d. Keratitis

B noninfectious endophthalmitis is an adverse event with several presenting etiologies, including retained lens material and other introduced toxic substances. frequency is unknown, but the occurrence is not rare. it is most often associated with TASS, an acute, rapid onset of sterile anterior segment inflammation that mimics infectious endopthalmitis, most commonly occurs after cataract surgery.

The IP receives a call from a young man who thinks he was exposed to HIV. He has just taken his first test (an enzyme-linked immunosorbent assay [ELISA]), which was negative. Which of the following is the most likely time frame after exposure in which HIV antibodies would be measurable in a blood test? a. 6 months b. 1 to 3 months c. 12 months d. 7 days

B people usually develop measurable levels of HIV antibodies within 30 days of infection, though some may take longer - up to 3 months in some cases. Before this happens, there is a period when antibody levels are too low to reliably be detected. This is called the "window period". It is during this time that an infected person can pass HIV to others but still have a negative result if given an antibody test

The IP is reviewing the facility's performance measures, which are used to benchmark against national data. The IP ensures that each performance measure includes which of the following characteristics: 1) Measure is reliable 2) Measure targets improvement in a health population 3) Measure is defined according to physician preference 4) Measure can be easily interpreted by the users of the data a. 1, 2, 3 b. 1, 2, 4 c. 2, 3, 4 d. 1, 3, 4

B performance measures focus on outcomes or processes and are used for internal improvement purposes, intra- or interorganizational comparisons, and by various external entities for making decisions about care. selection of performance measures for specific HAIs should focus on measures that have clear cut definitions, provide precise and usable information, are supported by prior studiess, and can be applied readily in most hospitals

The IP initiates a new program to encourage compliance with hand hygiene. On element to the program includes randomly distributing coupons for free coffee to employees who are seen adhering to hand hygiene recommendations. This is an example of which type of power? a. coercive b. reward c. legitimate d. expert

B power is an integral part of management and leadership. the give main types of power include coercive, expert, legitimate, referent, and reward. reward power refers to the ability to grant another person something that they desire or to remove or decrease things that the person does not desire.

What type of rate would the IP want to calculate to give feedback to the surgeons in the facility? a. procedure-specific b. provider-specific c. unit-specific d. device-specific

B providing feedback of appropriate SSI surveillance data to surgeons has been shown to be important in reducing SSI risk. furthermore, providing active rather than passive feedback of surveillance results to surgeons has the greatest effect in reducing SSI rates. when surgical teams are engaged in examining their SSI rates and in appraising clinical processes, there is greater probability of success in reducing infection rates

Which of the following is an example of surveillance on a process indicator? a. The incidence rate of Clostridioides difficile in the Bone Marrow Transplant Unit b. The rate of hand hygiene compliance in the Bone Marrow Transplant Unit c. The number of sharp object injuries in the month of May in the Bone Marrow Transplant Unit d. The prevalence of vancomycin-resistant enterococci (VRE) in the Bone Marrow Transplant Unit

B surveillance can be defined as "a comprehensive method of measuring outcomes and related processes of care, analyzing the data, and providing information to members of the healthcare team to assist in improving those outcomes" process indicators measure the program's activities and outputs. outcome indicators measure whether the program is achieving the expect effects/changes over time

What key infection control activity is defined as the systematic, ongoing collection, management, analysis, and interpretation of data followed by the dissemination of these data to public health programs to stimulate public health action? a. research b. surveillance c. benchmarking d. accreditation

B surveillance is defined as "ongoing collection, collation, and analysis of data and the ongoing dissemination of information to those who need to know so that action can be taken." surveillance is an essential component of an effective IP and control program. surveillance includes the collection of data with the ultimate objective of dissemination of that data to support and improve public health activities

A patient who was hospitalized for 2 days calls 3 days after discharge complaining that he has developed health care-associated scabies due to his recent inpatient stay. The IP knows that his scabies infestation is not health care-associated because: a. Scabies is only transmitted through contaminated linens, and the IP confirmed that all linens the patient came into contact with had been properly laundered b. the incubation period for scabies is longer than 5 days c. the incubation period for scabies is shorter than 3 days d. scabies is only transmitted through direct contact and none of the health care personnel who cared for the patient are infested

B the incubation period for scabies may be as short as 10 days but is typically between 4 and 6 weeks. Therefore, the patient could not have acquired scabies during the hospital stay because, based on the incubation period, he would need to have been exposed at least 5 days before he was admitted

As the sample size increases, how is the power of the study affected? a. power is independent of sample size b. power is increased c. power is decreased d. power approaches 0

B the power of a test is its ability to detect a specified difference (e.g. the probability of rejecting the null hypothesis when it is false). the power of a hypothesis test is affected by three factors: 1. sample size (n). In general, the greater the sample size, the greater the power of the test 2. significance level (a). the higher the significance level, the higher the power of the test 3. the "true" value of the parameter. the greater the difference between the "true" value of a parameter and the value specified in the null hypothesis, the greater the power of the test. That is, the greater the effect size, the greater the power of the test

The precision of an estimate of a relative risk depends on which of the following: a. generalizability b. size of the study c. validity of the study d. presence of bias

B the precision of the relative risk is related to the power of a study. statistical power is affected chiefly by the size of the effect and the size of the sample used to detect it.

An IP is conducting an educational session to help the nursing staff understand infectious disease transmission. She explains that the initial element in virulence is the ability of an organisms to survive in the external environment during transit between hosts. What is the second element of virulence? a. secretion of enzymes that enhance spread through tissues b. a mechanism for transmission to a new host c. invasion and dissemination in the host d. avoidance of host resistance

B the second element in virulence is a mechanism for transmission to a new host. for example, insect vectors may transmit pathogens by injecting material from salivary glands or defecation into sites of penetration on host skin. some viruses can survive and be transmitted to hands from environmental surfaces, such as bed rails. Some bacteria possess mechanisms of motility. when a microorganism reaches a favorable site for inducing disease, it must adhere to the structure that it will infect in order to survive

Plague is endemic in parts of the Southwest US. The word "endemic" means: a. natives are immune to plague b. an expected number of cases occurs each year in a given geographical area c. plague has become resistant to all forms of treatment for this population d. the disease is seen in a seasonal pattern each year for this area

B the term "endemic" refers to the usual incidence of a given disease within a geographical area during a specified time period

What is another name for crusted scabies?

B. Norwegian Scabies. scabies are parasitic mites on skin. most common symptoms is intense pruritus (itching). crusted scabies (Norwegian) presents as a crusty, scaly dermatitis usually on hands and feet. highly contagious due to number of mites present. itching remarkably minimal. usually in immunocompromised, elderly, disabled, or debilitates individuals. often appears as generalized dermatitis more widely distributed than the burrows, with extensive scaling and sometimes vesiculation and crusting; the usual severe itching may be reduced or absent

A measles exposure from a patient in a clinic was identified and an exposure workup was initiated. A staff exposure was defined as "nonimmune HCP with more than 5 minutes of same-room contact or face-to-face contact with the index patient." Forty-eight HCP were identified as possible exposures. Of these, 44 had documented immunity to measles. Of the remaining HCP, three did not have the same room or face-to-face contact. How many HCP were at risk of developing measles because of this exposure? a. 4 b. 45 c. 1 d. 48

C

After reviewing the quarterly report, the manager of the adult ICU contacts the IP for assistance to create a plan to reduce central line infections. Which of the following should the IP recommend: a. Wait for the next report to see if the rate has decreased b. Create an Intravascular Team c. Develop a multidisciplinary team to review and implement best practices d. Send a referral to Medical Affairs for peer review

C

An employee is exposed to a patient known to have chronic Hepatitis B. The employee is a known responder to the Hepatitis B vaccine, which was given to him as a student 5 years ago. What is the recommended postexposure treatment for the employee? a. Test the employee and all close personal contacts for Hepatitis B b. Start the hepatitis B series on the employee because of the length of time since previous vaccination c. No treatment is recommended for a known responder d. Recommend giving the employee the Hepatitis A vaccine

C According to the CDC guidelines, when the employee is known to have responded (converted) to positive Hep B antibody following immunization series, no treatment is recommended look up what a responder is. Just means vaccine worked?

A new employee who needs to be tested for TB infection before starting work has a history of BCG vaccination. Which method of TB testing would be the best choice in this situation? a. The TST would be the best method to use because it is the most cost-effective testing method b. A TST would be the best method to use because it distinguishes latent from active TB infection c. An interferon-gamma release assay (IGRA) blood test would be the best method to use because prior BCG immunization does not cause a false positive with this test d. An IGRA blood test would be the best method to use because it is a rapid test and provides results within 30 min

C An IGRA would be the best choice in this case because it will not be affected by the employee's prior BCG vaccination. Other advantages of IGRAs are that they do not require a follow-up visit to read the test and that the results are available within 24 hours. However, IGRA testing is generally more expensive than TST

The IP receives a call from a physician who is concerned that there is an outbreak of Acinetobacter baumannii, because he has cared for four patients in the past week who are infected with the organism. What is the IP's first step in responding to this call? a. Contact the lab to ask them to create an alert for any A. baumannii cases b. Contact hospital administration to request additional resources to investigate the outbreak c. Confirm that there is an outbreak by using her own surveillance data and lab records to compare the rates of Acinetobacter baumannii over the past year d. Initiate a case-control study to determine risk factors for A. baumannii infection

C CDC's primary components of the initial investigation include: -confirming the presence of an outbreak -identifying investigation team and resources -verifying the diagnosis -establishing a preliminary case definition -alerting administration and key healthcare partners about the investigation -performing a detailed literature review -developing a methodology for case finding -preparing an initial line list and epidemic curve -observing and reviewing potentially implicated patient care activities -considering whether environmental sampling or HCP sampling should be performed -implementing initial control measures -defining and refining the case definition -continuing case finding and surveillance -reviewing regularly control measures -maintaining surveillance activities -considering whether an analytic study should be performed

Which factor is commonly associated with c diff infection (CDI)? a. chemotherapeutic agents b. ACE inhibitors c. prophylactic antibiotics or antibiotics to treat a primary bacterial infection d. antiviral medication to treat a primary viral infection

C Cdiff is a spore-forming, gram-positive anaerobic bacillus that produces two exotoxins: toxin A and toxin B. it is a common cause of antibiotic-associated diarrhea (AAD), and it accounts for 15-24% of all episodes of AAD. the risk for disease increases in patients with: -antibiotic exposure -proton pump inhibitor exposure -GI surgery or manipulation -long length of stay in health care settings -a serious underlying illness -immunocompromising conditions -advanced age

The range of the correlation coefficient is a. -1 to 0 b. 0 to 1 c. -1 to 1 d. none of the above

C Correlation is used to calculate the direction and magnitude of a relationship between two variables. Correlation calculates a value, r, that measures the degree of the relationship. The calculated values can range between +1 and -1. The closer r is to +/- 1, the stronger the relationship. A positive correlation exists when one variable increases and causes the other to increase as well (eg the longer the urinary catheter is in place, the greater the risk of developing a UTI). a negative correlation occurs when one variable increases and causes the other to decrease (eg, increased hand washing results in fewer infections). the association between two variables decreases as r approaches 0. With a value of 0, there is no correlation

Types of tissue consider at high risk for CJD include: 1) Heart 2) Blood 3) Eye 4) Brain a. 1, 4 b. 1, 3 c. 3, 4 d. 2, 4

C Creutzfeldt-Jakob disease is a prion disease occurring in two forms: sporadic type and familial type due to genetic mutation. introduction of prion-laden tissue into the brain or body of an otherwise health patient or by use of contaminated medical equipment of invasive procedure. this condition has occurred when tissue transplants (dura mater, pericardium, and cornea) have been taken from patients with unsuspected CJD at time of death and used as donor organs.

A urine specimen collected from an indwelling urinary catheter was sent to the lab for culture and sensitivity testing. Culture results reported a colony count of 50,000 CFU/mL of E. coli. Sensitivity testing reported resistance to cephalosporin and sensitivity to ciprofloxacin. This organism is an example of: a. Methicillin resistance b. Aminoglycoside resistance c. Extended-spectrum beta-lactam (ESBL) resistance d. Quinolone resistance

C ESBLs are B-lactamases found in common gram-negative bacteria, such as E. coli and Klebsiella pneumoniae, which confer resistance to all B-lactam drugs except the carbapenems. Klebsiella species and E. coli are the most common ESBL-producing pathogens. ESBLs are enzymes that mediate resistance to extended-spectrum (third generation) cephalosporins and monobactams but do not affect cephamycins or carbapenems. The presence of an ESBL-producing organism in a clinical infection can result in treatment failure if one of the above classes of drugs is used. ESBLs can be difficult to detect because they have different levels of activity against various cephalosporins.

A nurse is concerned that a patient in the neurology ward has a prion disease after receiving a lab report stating that the patient had a positive nucleic acid test for John Cunningham virus (JCV) in the cerebrospinal fluid. What is the best response to give this nurse? a. the test is positive for Creutfzfeldt-Jakob disease, which is a prion disease b. the test is positive for Campylobacter jejuni, which is not a prion disease c. the test is positive for JCV (a polyomavirus), which is not a prion disease d. the test is positive for JCV (a polyomavirus), which is a prion disease

C JCV is a polyomavirus that can cause a demyelinating disease called progressive multifocal leukoencephalopathy in immunocompromised individuals. Because of the name and some presenting symptoms, JCV is sometimes confused with Creutzfeldt-Jakob disease (CJD), which is a prion disease

Because there is no vaccine for Hepatitis C, there have been national recommendations for prevention and control of Hepatitis C virus (HCV) infections. These include all but which recommendation? a. Screening and testing of blood donors b. Risk-reduction counseling and screening of persons at risk for Hepatitis C infection c. A national registry for all health care personnel known to be Hepatitis C antibody positive d. Adherence to Standard Precautions and safe work practices in health care settings

C No vaccine against HCV infection exists. National recommendations for prevention and control of HCV infection, issued in 1998, emphasize primary prevention activities to reduce the risk for HCV transmission. These activities include screening and testing of blood donors, viral inactivation of plasma-derived products, risk-reduction counseling and screening of persons at risk for HCV infection, and adherence to Standard Precautions and safe work practices in health care settings.

Which of the following statements about tuberculosis (TB) and airborne diseases among homeless individuals is most accurate? a. most TB infections among homeless individuals are reactivations of established disease b. sputum testing detects more than 90 percent of patients with TB c. screening for TB with chest X-ray may be the most cost-effective approach d. directly observed therapy in the acute hospital setting is associated with the highest completion rates

C TB incidence is higher in homeless populations than in the general population. molecular epi studies indicate that most TB cases occurring in the homeless are primary infections. the spread of TB among the homeless is related to recent person-to-person transmission, which produces outbreaks with large clusters in which more than 50 percent of persons are infected. homeless shelters are major sites of transmission. screening by chest radiography either periodically in all residents or specifically in symptomatic persons (e.g., chronic coughers) appears to be the most cost-effective approach for TB detection and diagnosis in this population

Which is true about a tuberculin skin test (TST) a. Positive TST indicates active TB infection b. Negative TST rules out active TB infection c. Positive TST indicates past exposure to TB d. Negative TST indicates past exposure to TB

C TST involves injection of purified protein derived from the mycobacterial cell wall. the test relies on the fact that persons who have been infected with TB will have a delayed-type hypersensitivity reaction to this reagent. The TST is a screening tool to detect people with latent TB infection (LTBI); it cannot be used for ruling in or ruling out active TB. This is because a positive TST merely indicates a history of LTBI at some time in the past--it conveys no information regarding the current status of the person's infection (which may have been previously cured). Likewise, a negative test does not rule out active TB because people with active tuberculosis may well have a negative TST, even in the presence of positive controls. In fact, up to 20 percent of persons with active TB will have negative TST results.

To calculate the catheter-associated urinary tract infection (CAUTI) rate for a unit for one month, the denominator should be: a. The number of patient days for the unit for the month b. The number of admissions for the unit for the month c. The number of patients with urinary catheters for the month d. The number of urinary catheter insertions for the month

C The CAUTI rate per 1000 urinary catheter days is calculated by dividing the number of CAUTIs by the number of catheter days and multiplying the result by 1000.

When an error does not result in an adverse event for a patient because the error was caught, it is called a: a. adverse event b. no harm event c. near miss event d. error report

C a near miss event is an unplanned event that could have resulted in injury, illness, or damage but did not, either by change or through timely intervention

Which of the following is not proof of measles immunity for health care personnel? a. documentation of vaccination with two doses of live measles virus-containing vaccine b. lab evidence of immunity c. born after 1957 d. lab confirmation of disease

C according to the ACIP, the following are proof of measles immunity for health care providers: 1) documentation of vaccination with two doses of live measles virus-containing vaccine, 2) lab evidence of immunity, 3) lab confirmation of disease, or 4) born before 1957. The latest recommendations also state that for unvaccinated personnel who were born before 1957 and who lack lab evidence of measles, rubella, or mumps immunity or lab confirmation of disease, health care facilities should consider vaccinating personnel with two doses of MMR vaccine at the appropriate interval (for measles and mumps) and one dose of MMR vaccine (for rubella), respectively

The Director of the Infection Prevention and Control Department has assigned one of her IPs to cofacilitate in a root cause analysis of an adverse event in collaboration with the Performance Improvement team. The IP plans to use process improvement tools and techniques during the analysis. Which of the following methods would best outline the possible causes of the event? a. Brainstorming b. Affinity diagrams c. Fishbone diagram d. Pareto chart

C affinity diagrams: gather large amounts of language data and creatively group the data based on lines of natural relationships. data usually collected from brainstorming or customer surveys. fishbone diagrams: map in detail the full range of paths and tasks that must be accomplished to achieve a primary or secondary goal. may help display the process of identifying contributing factor categories. pareto charts: series of vertical bars arranged and sorted in descending order of height from left to right with a cumulative percent line on the y-axis

The purpose of the antibiogram is to: a. Provide a monthly report on new and emerging antimicrobials b. Give IPs another metric to track c. Provide information on antimicrobial usage and resistance patterns in the community d. Give hospitals information needed for reporting data through the National Healthcare Safety Network (NHSN)

C an antibiogram simplifies multiple patients' antimicrobial sensitivity information at an institution into a single number for pathogens of interest in an effort to monitor trends emerging in drug resistance. help answer questions in two main areas: clinical care (what antimicrobial would be best to use in this hospital for this pathogen?) and infection prevention strategies (has the resistance of this pathogen to this antimicrobial changed in this hospital to warrant augmenting or diminishing our infection prevention interventions?)

The term for an extraneous variable that systematically varies with the independent variable and influences the dependent variable is a: a. predictor variable b. moderating variable c. experimental variable d. confounding variable

D a confounding variable is a variable that has an important confounding effect of the result is not among the variables being studied. it can suggest a false relationship between variables, or it can hide relationship that exists

The IP receives a call from the ER about a 38-year-old male with a 4-week history of cough and fever. He has just returned from a extended trip to the southwestern US. coccidioidomycosis is on the list of possible diagnoses. Which of the following is true regarding the epidemiology of coccidioidomycosis? a. Coccidioides spp are usually found at high elevations b. Coccidioides spp are usually found in wet climates c. Up to 50 percent of people in endemic areas have been exposed to Coccidioides spores d. Coccidioides spp are found on the surface of the soil

C coccidioidomycosis is a reemerging infectious disease caused by inhalation of airborne spores of the soil fungus Coccidioides immitis or C. posadasii. Coccidioides spp are native to the arid and desert areas in North America (California, Arizona, Texas, Utah, Nevada, New Mexico, and northern parts of Mexico), Central America, and South America. Coccidioides spp are found in lower elevation areas that receive less than 20 inches of rain per year and have warm, sandy soil. They are usually found 4 to 12 inches below the surface. Among persons living in coccidioides-endemic areas, approx 10 to 50 percent have been exposed to Coccidioides spores.

A patient has a nasal swab positive for methicillin-resistance Staphylococcus aureus (MRSA) in the absence of symptoms. This is an example of: a. normal flora b. colonization c. asymptomatic infection d. symptomatic infection

C colonization is the presence of microorganisms on skin, on mucous membranes, in open wounds, or in excretions or secretions in the absence of adverse clinical signs or symptoms

Which of the following indicates a strong positive correlation? a. r=0 b. r=-0.993 c. r=0.603 d. r=0.45

C correlation is a statistical technique that shows whether pairs of variables are related. correlation calculates a value, r, that measures the degree (strength) of the relationship. the calculated values can range between +1 and -1. the closer the r is to +/-1, the stronger the relationship. A positive correlation exists when one variable increases and causes the other to increase as well. A negative correlation occurs when one variable increases and causes the other to decrease. The association between two variables decreases as r approaches 0 (with a value of 0, there is no correlation)

Which organism found in food poisoning causes the most rapid onset of symptoms? a. Salmonella enteritidis b. Shigella sonnei c. Staphylococcus aureus d. Escherichia coli

C eating foods contaminated with toxins produced by S. aureus causes staphylococcal food poisoning. Food workers who carry Staphylococcus on their skin and then handle food without washing their hands contaminate foods by direct contact. The bacterium can also be found in unpasteurized milk and cheese products. Staphylococcus is salt tolerant and can grow in salty foods like ham. As the bacterium multiplies in food, it produces toxins that can cause food poisoning. Staphylococcal toxins are resistant to heat and cannot be destroyed by cooking. Foods at highest risk of producing toxins from Staph aureus are those that are made by hand and require no cooking. Some examples of foods that have caused staphylococcal food poisoning are sliced meat, puddings, pastries, and sandwiches. Staphylococcal toxins are fast acting, sometimes causing illness in as little as 30 minutes after eating contaminated foods, but symptoms usually develop within 1 to 6 hours. Patients typically experience several of the following: nausea, retching, vomiting, stomach cramps, and diarrhea

A microbe that can grow in the absence of oxygen but is also able to utilize oxygen for growth is a/an: a. aerobe b. obligate anaerobe c. facultative anaerobe d. microaerophilic aerobe

C facultative anaerobes will utilize oxygen if it is present but are still able to grow in the absence of oxygen. Microbes that must have oxygen to grow are called aerobes, whereas microbes that only grow in the absence of oxygen are called obligate anaerobes. microaerophilic bacteria need a small amount of oxygen for growth

The IP receives a call from the Food and Drug Administration (FDA) with an official request for private health information (PHI) about a patient who was admitted to the facility with botulism. How should the IP respond to this call? a. Tell the FDA officer that she cannot share PHI with the FDA due to Health Insurance Portability and Accountability Act (HIPAA) regulations b. Ask the FDA to contact the local health department to obtain information about the patient c. Provide the FDA officer with the minimum amount of information necessary related to the patient d. Transfer the call to the Risk Management Department

C for disclosures to a public health authority, covered entities may reasonably rely on a minimum necessary determination made by the public health authority in requesting the protected health information. for routine and recurring public health disclosures, covered entities may develop standard protocols, as part of their minimum necessary policies and procedures, that address the types and amount of protected health information that may be disclosed for such purposes

A newly hired infection prevention manager is addressing program deficits that occurred during the months the facility sought to fill the vacant position. the new manager must focus on many specific tasks, including working closely with others to clarify roles and responsibilities. This type of management approach is known as: a. charismatic b. situational c. functional d. motivational

C functional management is the most common type of organizational management. a functional manager is a person who has management authority over an organizational unit-such as a department-within a business, company, or other organization. functional managers have ongoing responsibilities and are not usually directly affiliated with project teams, other than ensuring that goals and objectives are aligned with the organization's overall strategy and vision.

An IP is updating the organization's infection prevention plan, which includes writing clearly stated goals and objectives. Which of the following statements might she consider including? a. vaccinate employees and volunteers for influenza every year b. serve as leader for facility safety rounds as needed c. achieve a 20 percent improvement in hand hygiene practice in the emergency department within 30 days d. collaborate with the laboratory to improve turnaround time for culture results

C goals are statements about general aims or purposes that are broad and long-range intended outcomes and concepts. Specific measurable objectives, however, describes the desired learning outcomes. Answer C is an example of a specific, measurable objective and includes a time component

Which of the following veins, when used for catheter insertion, has been associated with a lower incidence of central line-associated bloodstream infection (CLABSI)? a. Brachial b. Femoral c. Internal jugular d. Subclavian

D according to the CDC guidelines for the prevention of intravascular catheter-related infections, a subclavian site, rather than a jugular or femoral site, should be used in adult patients to minimize infection risk for nontunneled CVC placement. Use of the femoral vein for central venous access in adult patients should be avoided

A number of research studies have examined the relationship between nurse staffing levels and the risk of HAIs in the hospital settings. Which of the following statements has not been supported by the literature? a. Patients in an intensive care unit (ICU) with lower levels of nurse staffing had an increased risk for ventilator-associated pneumonia b. the use of nonpermanent staff significantly increases a patient's infection risk c. a specific evidence based nurse staffing level benchmark has been determined that is associated with decreased risk for HAI d. there is a relationship between adequate numbers of direct care providers (nurses) and the likelihood that CDC guidelines will be followed

C hospitals with low nurse staffing levels tend to have higher rates of poor patient outcomes such as pneumonia, shock, cardiac arrest, and urinary tract infections. Furthermore, a number of researchers have found the level and/or the use of nonpermanent staff also significantly increases a patient's infection risk. despite these data, determination of a specific evidence-based nurse staffing level benchmark that is associated with decreased risk for HAI has not been determined

Which of the following statements best describes human factors? a. ability to identify the many and various factors that have an impact upon a complex situation or event b. prevention of errors and adverse effects to patients associated with health care use c. environmental, organizational, and job elements and human and individual characteristics that influence behavior at work in a way that can affect health and safety d. the attitudes, beliefs, perceptions, and values that employees share in relation to safety

C human factors examine the relationship between people, the tools and equipment they use in the workplace, and the systems with which they interact. the goal of human factors is to minimize errors by focusing on improving efficiency, creativity, productivity, and job satisfaction. The application of human factors knowledge to health care can help design processes to improve patient safety.

The Director of Infection Prevention and Control is leading a process improvement project to decrease the rates of central line-associated bloodstream infections (CLABSI) in one of the hospitals ICUs. The multidisciplinary team has discussed multiple process improvement strategies to decrease these bloodstream infections. In developing the final improvement plan which of the choices below is most likely to help decrease the rates of these infections? a. performing a gap analysis each month b. performing a failure mode effect analysis (FMEA) immediately c. incorporating the use of a CLABSI bundle and a checklist to ensure that all aspects of the plan are followed d. perform a strengths, weaknesses, opportunities, and threats (SWOT) analysis

C implementing a formalized process reduces errors caused by lack of information and inconsistent procedures. Checklists and best practice bundles can promote process improvement and increase patient safety. by applying checklists to the prevention of infection within an organization and using simple steps such as washing hands and cleaning the skin with antiseptic, organizations can eliminate hazards and problems that affect patients every day. Bundles can create standardized and simplified processes and procedures, and the checklist provides an organized way to incorporate best practices

Which of the following is not a mechanical barrier to infection? a. intact skin b. mucous membranes c. secretions d. cilia

C mechanical barriers include skin, mucous membranes, and tears. the skin forms a physical barrier that is very impermeable to most infectious agents. movement due to cilia or peristalsis helps to keep air passages and the GI tract free from microorganisms. the trapping effect of mucus that lines the respiratory and GI tract helps protect the lungs and digestive systems from infection. chemical factors include fatty acids in sweat that inhibit the growth of bacteria. lysozyme and phospholipase found in tears, saliva, and nasal secretions can break down the cell wall of bacteria and destabilize bacterial membranes. the low pH of sweat and gastric secretions prevents growth of bacteria

The IP is teaching nurses how to assess infection risk in patients. Depletion of what cell type provides the best indication of susceptibility to most bacterial infections? a. Monocyte b. Eosinophil c. Neutrophil d. Lymphocyte

C neutrophils are the most abundant (40 to 75 percent) type of white blood cell and are formed from stem cells in the bone marrow. They are short-lived and highly motile. Neutrophils may be subdivided into segmented neutrophils (or segs) and banded neutrophils (or bands). They form part of the polymorphonuclear cell family together with basophils and eosinophils. Neutropenia is a granulocyte disorder characterized by an abnormally low number of neutrophils. Neutrophils serve as the primary defense against infections.

On July 7, a 30-year old male is admitted to the medical ICU with a 2-day history of acute gastroenteritis symptoms. The IP suspects the patient is infected with norovirus. Which of the following statements regarding the epidemiology of norovirus infection supports this? a. most norovirus outbreaks are caused by genotype GII.2 b. severe cases of norovirus are most common in adults aged 25 to 45 c. noroviruses are the most common cause of epidemic gastroenteritis worldwide d. norovirus outbreaks occur most commonly in the summer months

C noroviruses (NoVs) are the most common cause of epidemic gastroenteritis worldwide and the leading cause of foodborne outbreaks in the US. severe disease associated with NoV occurs most frequently among older adults, young children, and immunocompromised patients. NoV outbreaks occur year-round, but activity increases in the US during the winter months; 80 percent of reported outbreaks occur during Nov-April. Most NoV outbreaks are attributed to genotype GII.4, which evolves rapidly over time

Which statement about organizational conflict is correct? a. conflict resolution should focus on people, not issues b. openness and transparency by management leads to conflict c. conflict is a natural process within systems and fosters a search for alternatives d. conflict is an immediate sign of dysfunctional work teams

C organizational conflict is a state of discord caused by the actual or perceived opposition of needs, values, and interests between people working together. Organizational conflict stimulates a search for alternatives and can represent an opportunity for productive change. Acknowledging the existence of the conflict and investigating the source of it can lead to creative solutions

Anaerobic cultures should be used for any of the following sites except: a. Blood b. Transtracheal aspirate c. Spinal fluid d. Sputum

D an anerobic bacteria culture is a method used to grow anaerobes from a clinical specimen. Anaerobes are commonly found on mucous membranes and other sites such as the vagina and oral cavity. Therefore, specimens likely to be contaminated with these organisms should not be submitted for culture. Specimens that are not suitable for anaerobic cultures include: -sputum -rectal swab -nasal or throat swab -urethral swab -voided urine

Which of the following describes the correct storage to maintain sterility of reprocessed items? a. At least 12 inches below the ceiling, at least six inches above the floor, at least one inch from the outside wall, and on a rack with a solid bottom b. At least 18 inches below the ceiling, at least six inches above the floor, at least one inch from the outside wall, and on a rack with a solid bottom c. At least 18 inches below the ceiling, at least eight inches above the floor, at least two inches from the outside wall, and on a rack with a solid bottom d. At least 18 inches below the ceiling, at least eight inches above the floor, at least one inch from the outside wall, and on a rack with a solid bottom

C physical storage restrictions are recommended to support an environment that is conducive to maintaining the sterility of reprocessed items. sterilized items should be stored far enough away from the floor, the ceiling, and outside walls to allow adequate air circulation, ease of cleaning, and compliance with fire codes. 18 inches below the ceiling or the sprinkler head, 8-10 inches above floor, 2 in from outside walls, on closed or covered shelves, packages are not bent or crushed, in limited traffic areas, away from wet locations, first item in is first item out, only on designated areas, with heavy instrument trays placed on middle shelves, wrapped packages are not stored under rigid sterilization

A preliminary microbiology report states that a patient's blood culture grew aerobic, Gram-negative bacilli. Which of the following is the most likely genus and species of the organism: a. Enterococcus faecalis b. Bacteroides fragilis c. Acinetobacter baumanii d. Neisseria meningitides

C psuedomonas spp, acientobacter spp, and other glucose nonfermentative bacilli are common causes of HAIs. notable for their intrinsic antimicrobial resistance. minimal growth requirements and cause a wide array of opportunistic infections

Coughing, sneezing, and talking are best associated with which form of disease transmission? a. Airborne transmission b. Direct contact transmission c. Droplet transmission d. Indirect contact transmission

C respiratory droplets carrying infectious pathogens transmit infection when they travel directly from the respiratory tract of the infectious individual to susceptible mucosal surfaces of the recipient. Transmission occurs when large droplets (greater than 5um) containing the infectious agent are propelled a short distance through the air (eg, by coughing, sneezing, or talking) and come into direct contact with conjunctivae or mucous membranes. When droplets land or infectious secretions are deposited on surfaces close to the patient, pathogens can be acquired indirectly by health care personnel.

Vaginal probes with probe covers require which type of disinfection: a. Low-level disinfection b. Intermediate-level disinfection c. High-level disinfection d. Sterilization

C semicritical items are those that come in contact with mucous membranes or nonintact skin. these medical devices should be free of all microorganisms, although small numbers of bacterial spores may be present. they minimally require high-level disinfection using chemical disinfectants (i.e. hydrogen peroxide)

A patient is admitted with pruritic lesions on the hands, webs of fingers, wrists, the extensor surfaces of the elbows and knees, and the outer surfaces of the feet, armpits, buttocks, and waist. The most likely diagnosis is: a. Scarlet fever b. Herpes zoster c. Scabies d. Measles

C skin infestations by the mite Sarcoptes scabiei var hominis are commonly known as scabies. Humans are the natural reservoir of S. scabiei var hominis. Mites are transmitted through direct contact with infested persons; less frequently, transmission may occur through contact with clothing or bedding (fomites). Spread of the mite to a different part of the body can occur by manual transfer or scratching. Approximately two-thirds of cases have burrow-type pruritic lesions on hands, webs of fingers, wrists, extensor surfaces of elbows and knees, as well as outer surfaces of feet, armpits, buttocks, and waist. Spread can also occur to arms, trunk, legs, penis, scrotum, and nipples

Which of the following bioterrorism agents has the highest fatality rate? a. Q fever b. Ricin c. Smallpox d. Severe acute respiratory syndrome (SARS)

C smallpox is an acute, contagious, and sometimes fatal disease caused by the variola virus (an orthopoxvirus) and marked by fever and a distinctive progressive skin rash. The majority of patients with smallpox recover, but death may occur in up to 30% of cases. Bioterrorism agents can be separated into three categories, depending on how easily they can be spread and the severity of illness or death they cause. Category A agents, which includes smallpox, are considered the highest risk to national security because they: -can be easily disseminated or transmitted from person to person -result in high mortality rates and have the potential for major public health impact -might cause public panic and social disruption -require special action for public health preparedness

In any normal distribution, the proportion of observations that are within two standard deviations of the mean is closest to: a. 0.50 b. 0.68 c. 0.95 d. 0.98

C standard deviation is a measure of dispersion of the raw scores that reflects the variability in values around the mean. the significance of the standard deviation is that with normal (bell-shaped) distributions, the following empirical rules apply for the normal curve: -the interval from one standard deviation below the mean to one standard deviation above the mean contains approximately 68 percent of the measurements -the interval from two standard deviations below the mean to two standard deviations above the mean contains approximately 95 percent of the measurements -the interval from three standard deviations below the mean to 3 above contains approx 99.7 percent of all measurements

Statistical process control (SPC) charts are used for all of the following purposes except: a. monitor the process of care b. facilitate the determination of variation c. eliminate natural variation d. monitor outcomes

C statistical process controls (SPC) is a method of quality control that uses statistical methods and is an essential component of quality assurance and performance improvement . the principles of SPC are used to monitor both processes and outcomes in a systematic and statistically valid manner. control charts can assist in determining special-cause or common-cause variations, which may be helpful for early detection of abnormal events

What action is indicated when the IP is asked to help determine if a worker has experienced occupational acquisition of an infectious agent or disease in order to receive workers' compensation benefit? a. Provide enough information to prove or disprove the employee's claim b. Notify the facility's attorney immediately c. Review the workers' compensation system in place d. Perform a root cause analysis to investigate

C the IP may be asked to help assess a situation to determine if a worker has experienced occupational acquisition of an infectious agent or disease. the IP should be familiar with the workers' compensation systems in place within their state and facility. The US Department of Labor administers and enforces federal regulations related to workplace activities including workers' compensation

A hospital has hired a new manager of the Microbio section of the lab. during the initial discussion with the manager about the infection prevention and control program, the IP stresses the importance of collaboration between the departments in reducing health care-associated infections (HAI's). Of the choices below, which activity will best meet this goal? a. the microbio staff's compliance with the annual flu vaccination program and TB skin testing b. the microbio staff's participation in the periodic infection prevention educational sessions for hospital staff c. microbio's prompt notification to the IP and control department of any organism's unusual resistance pattern d. the microbio manager's attendance at local, state, and/or national infection prevention and control educational conferences

C the clinical microbio lab is an important partner in the practice of infection prevention. the active involvement and cooperation of the microbio lab is critical to the functioning of the infection control program, particularly in the surveillance and the use of lab services for epi purposes. surveillance requires high-quality lab data that are timely and easily accessible. the CDC recommends that all health care orgs establish systems to ensure clinical microbio labs promptly notify infection control staff or a medical director/designee when a novel resistance pattern for the facility is detected

Which of the following is not an infection prevention objective of an occupational health program? a. Contain costs by preventing infectious diseases that result in absenteeism and disability b. Provide care to personnel for work-related illnesses or exposures c. Educate patients about the principles of infection prevention d. Collaborate with the Infection Prevention Department in monitoring and investigating potentially harmful infectious exposures and outbreaks

C the elements of an infection prevention OHP include: 1) education of HCP about the principles of infection prevention and their individual responsibility for infection prevention 2) collaboration with the IPC department in monitoring and investigating potentially harmful infectious disease exposures and outbreaks 3) provision of care and follow up to HCP to work-related illnesses, injuries, and exposures 4) identify work-related infection risks and institute appropriate preventive measures 5) contain costs by preventing infectious diseases that result in absenteeism and disability

The occupational health nurse has requested the IP's assistance in reporting the nursing needlestick rate annually. Which formula should be used? a. Total number of needlesticks reported by nursing divided by the average daily census b. Total number of needlesticks reported by nursing divided by the needle devices used by nursing c. Total number of needlesticks reported by nursing divided by the number of full-time nurses employed during the year d. Total number of needlesticks reported by nursing divided by the number of injections given by nurses

C the rate of needlestick injuries can be divided by occupational category. For example: total number of needlesticks reported by nursing in 1 year divided by number of fill-time equivalent nurses employed in that year equals the rate of needlesticks per full-time equivalent nurse per year

An IP is participating on a multidisciplinary team formed to decrease sharps injuries in an Ambulatory Surgical Center. Of the following possible activities, which would be most likely to assist the team? a. A quarterly review of sharps injury data stratified by surgical team b. An analysis of employee participation in the Hep B vaccination program c. Root cause analysis after exposure incidents d. A review of surgery duration in cases in which sharp injuries were reported

C the root cause analysis process takes a retrospective look at adverse outcomes and determines what happened, why it happened, and what an organization can do to prevent the situation from recurring. When conducting a root cause analysis, a multidisciplinary team discovers basic and contributing causes for what happened. The entire process identifies changes to a particular process or system that improves safety or reduces process error. Performing a root cause analysis is the best approach to take to accomplish the goal of decreasing sharp injuries.

All of the following are true and should be communicated to patients before administering influenza vaccination except: a. the vaccine is formulated annually to protect against influenza strains likely to circulate in the US in the upcoming winter b. Inactivated influenza vaccine contains noninfectious viral components and cannot cause influenza c. Patients who received specific antivirals (ie, acyclovir, famciclovir, or valacyclovir) within the preceding 24 hours should not receive influenza vaccination d. Fever, malaise, myalgia, and other systemic symptoms can occur after vaccination

C there is no contraindication for patients receiving the influenza vaccine if they received specific antivirals within the preceding 24 hours. However, precautions must be taken when administering the zoster vaccine to patients who have received antivirals 24 hours before vaccination; use of these antiviral drugs should be avoided for 14 days after zoster vaccination. Before administering the influenza vaccine, patients should be informed of the following: 1. the vaccine is formulated annually to protect against influenza strains likely to circulate in the US in the upcoming winter 2. Inactivated influenza vaccine contains noninfectious viral components and cannot cause influenza. Live attenuated influenza vaccine (LAIV) can cause nasal congestion, sore throat, and headache for a few days 3. Other respiratory diseases unrelated to influenza vaccination can occur after vaccination 4. Fever, malaise, myalgia, and other systemic symptoms can occur after vaccination, especially in persons with no prior exposure to influenza vaccine (eg young children). However, in placebo-controlled studies, rates were similar between vaccine and placebo recipients 5. Influenza vaccine should not be administered to persons known to have anaphylactic hypersensitivity to eggs or other components of the vaccine without first consulting a physician

Western blot testing for HIV is used to detect: a. HIV DNA in a serum sample b. HIV RNA in a serum sample c. HIV antibodies in a serum sample d. HIV proteins in a serum sample

C western blot is a confirmatory test that is performed on all samples that have tested positive for HIV on an enzyme immunosorbent assay (EIA). HIV proteins are run through a gel matrix and then transferred to a thin membrane. The membrane is incubated with a serum sample, washed, and then incubated with a secondary antibody. if antibodies to the HIV proteins that were run through the gel matrix are present in the serum sample, they will compound with those proteins and will not be washed away. a secondary antibody with fluorescent tag will then bind to the HIV antibody-protein compound, and this allows the antibody-protein compounds to be visualized

The bacterium most likely to be transmitted from mother to infant during labor and cause neonatal sepsis is: a. Escherichia coli b. Staphylococcus aureus c. Group B streptococcus d. Group A streptococcus

C. Group B Streptococcus Group B streptococci are common inhabitants of the lower GI and genital tracts. GBS invasive infection in neonates typically occurs within the first week of life where the infection is transmitted from the colonized mother. manifestations include pneumonia, sepsis, meningitis, and bone/joint infection. risk factors for GBS infection in infants born to colonized mothers include preterm delivery, prolonged membrane rupture during labor, maternal fever during labor, and maternal history of prior infant with GBS sepsis.

In 1997, the joint commission on accreditation of healthcare organizations (TJC) mandated the use of root cause analysis to: a. document instances of medical malpractice b. predict the occurrence of an incident c. improve staffing issues d. investigate sentinel events in accredited hospitals

D a sentinel event is defined by TJC as any unanticipated event in a health care setting resulting in death or serious physical or psychological injury to a patient or patients that is not related to the natural course of the patient's illness. sentinel events specifically include loss of a limb or gross motor function and any event for which a recurrence would carry a risk of a serious adverse outcome. sentinel events are identified under TJC accreditation policies to help aid in root cause analysis and to assist in development of preventative measures

Which of the following is not evidence of varicella immunity in health care personnel? a. Evidence of two doses of the varicella vaccine d. Lab evidence of immunity c. Lab confirmation of disease d. Born before 1980

D According to the ACIP, the following are proof of immunity for varicella in a health care provider: 1) evidence of two doses of the varicella vaccine, 2) lab evidence of immunity to varicella, or 3) lab confirmation of disease. Birth before 1980 should not be considered proof of immunity for health care providers.

The IP is reviewing the immunization records of health care personnel at their facility and discovers that employees born before 1957 do not have any record of receiving the MMR vaccine. What should she recommend to the HR director regarding employees born before 1957? a. They are considered immune and do not require follow-up b. They should receive two doses of the vaccine 4 weeks apart c. They are only required to provide proof of immunity to measles d. They are required to provide proof of immunity to measles, mumps, and rubella

D All persons working in health care facilities should be immune to measles, mumps, and rubella. It is reasonable to require proof of immunization. Proof of immunity consists of documented vaccination with one or more doses of live vaccine on or after the first birthday, laboratory evidence of immunity to these diseases, or history of these diseases based on a health care worker's diagnosis. Although birth before 1957 is considered acceptable evidence of measles, rubella, and mumps immunity, ACIP recommends that health care facilities consider vaccinating unvaccinated personnel before 1957 who do not have lab evidence of MMR immunity; lab confirmation of disease; or vaccination with two appropriately spaced doses of MMR vaccine for measles and mumps or one dose of MMR vaccine for rubella.

Which of the following statements is true regarding CVS? a. anticoagulant therapy can reduce the risk of catheter-related infection b. positioning at the insertion site minimizes catheter tip malposition c. the CVC should be sutured in place d. stopcocks can increase the contamination rate

D according to the CDC guidelines for the prevention of intravascular catheter-related infections, a CVC with the minimum number of ports or lumens essential for the management of the patient should be used. Stopcocks used for injection of medications, administration of IV infusions, and collection of blood samples represent a potential portal of entry for microorganisms into vascular access catheters and IV fluids. Stopcocks should be capped when not being used. In general, closed catheter access systems are associated with fewer catheter-related infections than open systems and should be used preferentially

There has been a local bioterrorism event and three health care personnel were exposed to inhalation anthrax. They have been decontaminated and are taking PEP, and they would like to return to work. The incubation period of inhalation anthrax is usually about 7 days but can be as long as 2 months. What should the IP's recommendation be regarding work restrictions for these employees? a. They will not be allowed to return to work for the duration of the 2-month incubation period b. They ill not be allowed to return to work for the duration of prophylactic treatment c. They may return to work but must wear respiratory protection while in the facility d. They may return to work with no restrictions

D Although work restrictions in such a scenario would likely be influenced by outside agencies such as the local and state health departments and the CDC, inhalation anthrax is not transmissible among humans, so the exposed health care workers pose no risk to patients or other employees.

A gram-negative bacterium that is responsible for chronic antral gastritis and is a major factor in peptic ulcer disease is: a. Streptococcus pyogenes b. Salmonella typhi c. Clostridioides difficile d. Helicobacter pylori

D H pylori is a gram-negative bacterium that causes chronic gastritis in humans. This bacterium is also considered to be a common cause of ulcers worldwide: as many as 90 perfect of people with ulcers have detectable H pylori. It is also linked to the development of duodenal ulcers and stomach cancer. However, more than 80 percent of individuals infected with the bacterium are asymptomatic, and it has been postulated that it may play an important role in the natural stomach ecology

When a normal distribution is graphed, which of the following are true? 1) There is a continuous, symmetrical distribution in which both tails extend to infinity 2) the mean, median, and mode are identical 3) 68.3% of the area lies between the mean and +/-2 standard deviations 4) the shape of the curve is determined by the mean and standard deviation a. 1, 2, 3 b. 2, 3, 4 c. 1, 3, 4 d. 1, 2, 4

D If the distribution (spread) of the values is even on both sides of the mean (both halves are equal), it is a normal distribution. Properties of a normal distribution include: -forms a symmetric bell-shaped curve -50 percent of the scores lie above and 50 percent below the midpoint of the distribution -the population clusters around a central point and then trails off symmetrically in both directions with fewer and fewer large and small individuals at the upper and lower ends, respectively -mean, median, and mode are located at the midpoint of the x axis

The US Public Health Service's Advisory Committee on Immunization Practices (ACIP) recommends all of the following immunizations be provided to health care personnel except: a. Hepatitis A and B vaccines b. Influenza vaccine c. Measles, mumps, and rubella (MMR) and varicella-zoster vaccines (if not immune) d. Bacillus Calmette-Guerin (BCG)

D Immunization programs provide protected from vaccine-preventable diseases for both the workers and those under their care. The infectious diseases for which vaccines are available for pre-exposure intervention include Hepatitis A and B, influenza, measles, mumps, rubella, tetanus, pertussis, and varicella-zoster (chickenpox)

The infection preventionist (IP) is assisting Employee Health with personnel tuberculosis (TB) skin testing. Which of the following represents a known tuberculin skin test (TST) conversion in a health care worker? a. Prior tuberculin test results are not available, but the current result is 16mm after 48 hours b. Tuberculin reaction 1 year ago was 9mm, and the current result is 13mm c. A prior tuberculin reaction was not measured, but the employee states it was dime-sized. The current result is 11mm d. Tuberculin reaction 1 year ago was 3mm, and the current result is 18mm

D Interpretation of the TST depends on measured TST induration in millimeters, the person's risk for being infected with M. tuberculosis, and risk for progressive to active TB if infected. The TST test should be interpreted according to the CDC guidelines. A health care worker without known exposure who demonstrates an increase of >= 10mm is considered a positive result.

The IP is reviewing the history of a patient who has been in the facility on a ventilator for 1 week. All of the following are risk factors for colonization and infection with multidrug-resistance pathogens except: a. antimicrobial therapy in preceding 90 days b. current hospitalization of 5 days or more c. immunosuppressive state or therapy d. low frequency of antibiotic resistance in the facility

D Pneumonia may be caused by a wide variety of pathogens, but multidrug-resistant organisms (MDROs) are becoming more relevant as etiologic agents. Risk factors for colonization and infection with MDRO are as follows: -antimicrobial therapy in preceding 90 days -current hospitalization of 5 days or more -high frequency of antibiotic resistance in the community or in the specific hospital unit -presence of risk factors for health care-associated pneumonia -immunosuppressive state or therapy

The new IP for a long-term facility is assessing adherence to the facility's hand hygiene policies. Which of the following should he include when reporting his findings? a. The number of hand hygiene episodes performed by personnel divided by the volume of soap used in the facility b. the number of hand hygiene episodes performed by personnel divided by the number of patient days times 1000 c. the number of hand hygiene episodes performed by personnel divided by the volume of alcohol-based hand rub d. the number of hand hygiene episodes performed by personnel divided by the number of hand hygiene opportunities

D The CDC guideline and the Joint Commission require that health care personnel adherence to recommended hand hygiene policies be monitored and that health care personnel be provided with information about their performance. Acceptable methods for measuring hand hygiene adherence include: -periodically conduct an observational study to determine the rate of adherence (number of hand hygiene episodes performed/number of hand hygiene opportunities) by ward or service. In addition to monitoring the rate of adherence, facilities may also assess the quality of hand hygiene adherence (time spent per hand hygiene episode, whether soap was used, etc.) -monitor the volume of specific hand hygiene products (e.g. soap, hand rub, hand lotion) used per 1000 patient days -monitor adherence to artificial fingernail policies

Urinary tract infections in the postpartum period have three important risk factors. Which of the following is not one of them: a. Induction of labor b. Tocolysis c. Cesarean delivery d. Twin births

D Tocolysis is an obstetrical procedure to prolong gestation in patients, some of which are experiencing preterm labor.

The Employee Health Service has notified the IP that seven employees have P aeruginosa folliculitis. Initial investigation reveals that six of the seven cases belong to the same health club. Working on the hypothesis that the whirlpool at the health club is associated with the infections, the IP decides to conduct a case-control study using two controls for each case. Which of the following groups is the most appropriate control? a. non-ill family members for the ill employees b. non-ill hospital employees matched for age and sex c. hospitalized patients with P. aeruginosa folliculitis matched for age and sex d. non-ill members of the health club matched for age and sex

D case-control studies begin with the identification of persons who have the outcome of interest. Then a control group of individuals without the outcome is selected for comparison. the selection of an appropriate control group is critical in that control patients must not only have the outcome of interest but also should be similar to the cases in the potential for exposure during the period of risk being evaluated. controls are matched to cases on one or more attributes (i.e. age, gender, smoking status, etc.). Each case/control pair then has identical values on the matching factors. Therefore, the most appropriate controls would be non-ill members of the health club.

The most common organism associated with pneumonia in school-aged children and young adults is: a. Neisseria meningitidis b. Streptococcus pneumoniae c. Staphylococcus aureus d. Mycoplasma pneumoniae

D common causes of bacterial pneumonia are Streptococcus pneumoniae (pneumococcus) and, especially in kids, Mycoplasma pneumoniae

Which of the following is an example of the principle of emergency management called "mitigation?" a. Implementation of the hospital's emergency management plan during a hurricane b. Recovery efforts after a major flood has subsided c. A facility-wide bioterrorism disaster drill d. Funding a program that will provide ring vaccination of exposed people against smallpox during an outbreak

D disasters should be planned and responded to using the principles of emergency management: mitigation, preparedness, response and recovery mitigation describes actions taken to decrease the potential impact of a situation. include interventions to either prevent or reduce morbidity and mortality and ease the economic and social impact of the event on the affected community. can range from new policies or laws related to the use of smallpox vaccine, putting shutters on houses in areas at risk for hurricanes, elevating houses in floodplains

Which of the following is not an effect of malnutrition on the body's immune system? a. intestinal bacteria may be altered b. tissue integrity is impaired c. mucosal secretions are decreased d. urine may be colonized with bacteria

D effects of protein/calorie malnutrition on immune function. Barrier functions -tissue integrity is impaired: decreased intestinal epithelium/mucosal lining, atrophy of the gut's associated lymphoid tissue (part of the immunologic barrier of the intestine) -respiratory tract cilia are decreased -mucosal secretions are reduced -intestinal bacteria may be altered; colonization in the normally sterile upper small bowel may occur -acid secretion may be decreased in the stomach

Staff assisting with bronchoscopy of a patient with suspected TB must wear which type of respiratory protection? a. surgical or procedure mask b. face shield c. protection is not required d. a fit-tested respirator or powered air purifying respirator (PAPR)

D health care personnel assisting with bronchoscopy should wear appropriate personal protective equipment including a fit-tested respirator or a PAPR

The most common reservoir for highly pathogenic avian influenza H5N1 virus is: a. migratory birds b. pigeons and doves c. waterfowl d. domestic poultry

D highly pathogenic avian influenza A (H5N1) virus is a virus that occurs mainly in birds, especially domestic poultry. H5N1 is highly contagious among birds and can be deadly to them. though relatively rare, sporadic human infections with this virus have occurred and have caused serious illness and death.

The IP is asked to recommend the length of time a staff member who has developed influenza should be excluded (furloughed) from work duties. The staff member was diagnosed with influenza on March 15. She consults the CDC infection control guidance for the prevention and control of influenza in acute care facilities and recommends that the employee should: a. Remain off work until March 20 b. Remain off work for the duration of the illness c. Remain off work until March 21 d. Remain off work for 5 days (March 20) or until symptoms have resolved, whichever is longer

D influenza is primarily spread between individuals via respiratory secretions (droplet spread). Viral shedding starts 24 to 48 hours after infection, and typically 24 hours before the onset of symptoms. Shedding normally persists less than 5 days but can be longer in children and in immunocompromised persons. Thus, adults are typically infectious from the day before symptoms begin until approximately 5 days after the onset of illness. Staff members who develop influenza illness should be furloughed for 5 days after diagnosis or the duration of their illness, whichever is longer.

Which of the following statements is false regarding influenza viruses? a. They are divided into three categories: A, B, and C b. influenza A strains have been the predominant cause of worldwide epidemics (pandemics) c. influenza A and B strains have been named according to the city or state and year of their initial isolation d. influenza B stains have not been associated with large epidemics

D influenza viruses are divided into three categories: A, B, and C. All three contain negative-sense, segmented, single-strand RNA molecules. Influenza A viruses are the most common clinical isolates, and they are subdivided by differences in two surface proteins: hemagglutinin (H) and neuraminidase (N). Three H and two N antigenic subtypes account for virtually all human infections. The most prevalent influenza A strains in humans in the last 30 years have been H3N2 and H1N1. In recent years, influenza A and B strains also have been named according to the city or state and year of their initial isolation, for example, A/Texas/36/1991 (H1N1). Influenza B strains also cause epidemic disease, but the clinical illnesses tend to be milder than illnesses cause by influenza A. Influenza C strains are much less common (<1 percent of influenza infections), and they produce generally mild illness (usually a "common cold"). Influenza C strains have not been associated with a large epidemic.

Which of the following is not considered one of the criteria for causality: a. the incidence of disease is higher in those who are exposed to the factor b. evidence that the independent and dependent variables are related c. the association has been observed in numerous studies d. the onset of disease must precede exposure to the causal factor

D look at #35 in class notes for outline of all Hill's criteria

Lyme disease is commonly found in all of the following regions of the United States except: a. New England b. Mid-Atlantic c. Upper Midwest d. Southeast

D lyme disease is caused by the bacterium Borrelia burgdorferi and is transmitted to humans through the bite of infected black-legged ticks. Typical symptoms include fever, headache, fatigue, and a characteristic skin rash called erythema migrans. if left untreated, infection can spread to the joints, heart, and nervous system. In the US, lyme disease has been reported in all 50 states but occurs most commonly in three principal areas; New England and the Mid-Atlantic states, the upper Midwest with concentration in the Great Lakes region, and several counties in northern California

Many external forces can impact the performance of an infection prevention team. Which of the following is not an example of an external force? a. a mandate from the CEO to reduce costs by 8 percent b. changes in regulatory and accrediting standards c. increasing costs of supplies used for patient isolation d. department expenses to support IP certification

D managers must recognize and respond to all factors that affect their organizations. organizational change is driven through fluctuations in the internal and external environments. the external environment includes factors that occur outside of the department that cause changes within and are, for the most part, beyond the control of the department. common external factors include competition, the economy, technology, political and social conditions, and resources

What is the appropriate temperature for vaccines that require refrigeration? a. 46F to 55F (8C to 13C) b. 25F to 35F (-4C to 2C) c. 25F to 45F (-4C to 7C) d. 35F to 46F (2 to 8C)

D most routinely recommended vaccines should be stored in a refrigerator between 35F and 46F (2C and 8C) with a desired average temperature of 40F (5C). Exposure to temperatures outside this range may result in reduced vaccine potency and increased risk of vaccine-preventable diseases

All of the following are descriptions of patient immunocompromised status except: a. HIV with CD4 count <200 b. leukemia or lymphoma c. neutropenia (absolute neutrophils count <500/mm3) d. 1 year post-bone marrow transplant

D patients with immunocompromised status include those with the following: -neutropenia -leukemia or lymphoma -HIV with CD4 count <200 -splenectomy -early post-transplant -on cytotoxic chemotherapy -on high-dose steroid therapy

A robust Performance Improvement team should perform all of the following except: a. Observational audits b. Benchmark comparisons c. Root cause analyses d. House wide infection rates

D performance improvement teams increase problem solving and efficiency, raise morale and productivity, use integrative rather than imposed solutions, increase acceptance of the solution, and tap the potential in people and their fundamental knowledge of the process gap analysis: compare best practices with the current processes and determine the steps to take to move from a current state to desired future state root cause: retrospective look at adverse outcomes and determines what happened, why, and what an org can do to prevent the situation from recurring

The IP should recommend all of the following prevention measures for a pregnant influenza-infected patient during delivery except: a. during labor and delivery, the patient should wear a mask b. the patient should be placed on droplet precautions c. after the infant is born, the mother should wear a surgical mask and then practice hand hygiene before handling the baby d. all persons who come within 3 feet of the mother should wear a surgical mask and practice hand hygiene before and after contact with the mother

D pregnant women and infants are at increased risk of hospitalization from influenza complications. pregnant women with influenza in the L&D suite should be placed on droplet precautions; she does not need to wear a mask during the time of delivery. after the infant is born, the mother should put on a surgical mask and then practice hand hygiene before handling the baby. All persons who come within 3 feet of the mother should wear a surgical mask and practice hand hygiene before and after contact with the mother. All persons in the delivery room should practice hand hygiene before and after handling the baby.

The causative organism of Creutzfeldt-Jakob disease is a: a. Helminth b. Diphtheroid c. Spirochete d. Prion

D prion diseases or transmissible spongiform encephalopathies (TSEs) are a family of rare progressive neurodegenerative disorders that affect both humans and animals. They are distinguished by long incubation periods, characteristic spongiform changes associated with neuronal loss, and a failure to induce inflammatory response. The causative agents of TSEs are believed to be prions. The term "prions" refers to abnormal, pathogenic agents that are transmissible and able to induce abnormal folding of specific normal cellular proteins called prion proteins that are found most abundantly in the brain. The functions of these normal prion proteins are still not completely understood. The abnormal folding of the prion proteins leads to brain damage and the characteristic signs and symptoms of the disease. Prion diseases are usually rapidly progressive and always fatal

While the IP performs environmental rounds in the pharmacy, she observed a pharmacy technician using poor technique while working under the laminar airflow hood. She reviews the Pharmacy's policy for the frequency of training on aseptic technique for employees preparing sterile solutions. Which of the following is the best recommended frequency of infection prevention and control training for unlicensed employees who prepare sterile solutions? a. monthly until the employee can demonstrate proficiency by return demonstration b. every 6 months for 1 year after the date of hire c. annually on employee's anniversary date of hire with other training programs d. annually and whenever unacceptable technique is observed

D quality problems associated with compounded sterile and nonsterile pharmacy preparations have resulted in recalls, patient injury, and death. The American Society of Health System Pharmacists requires that all personnel be properly trained by the following means: -prior to commencing any compounding, perform thorough didactic instruction in the theory and practice of sterile preparations, with evaluation of technique annually (for low- and medium-risk level) and semiannually (for high-risk level) -compounder evaluations should include a formal written exam and practical evaluation of aseptic technique using growth media (media fills)

Specificity of a test for infection or disease is calculated as: a. The number of true negatives divided by the number of positives found, times 100 b. The number of true negatives divided by the total number of persons with disease, times 100 c. The number of true positives divided by the total number of persons with disease, times 100 d. The number of true negatives divided by the total number of persons without disease, times 100

D specificity (sometimes called the true negative rate) measures the proportion of negatives that are correctly identified as such (e.g., the percentage of healthy people who are correctly identified as not having the condition). Sensitivity (also called the true positive rate) measures the proportion of actual positives that are correctly identified as such (e.g. the percentage of sick people who are correctly identified as having the conditions). specificity = True negatives / (true negatives + false positives)

On a normally distributed data set, what percentage of values lies within three standard deviations from the mean? a. 68.2 b. 95.5 c. 92.4 d. 99.7

D standard deviation is a measure of dispersion of the raw scores that reflects the variability in values around the mean. it employs the squared deviations from the mean (variance), which therefore gives added emphasis to larger deviations. the standard deviation indicates how small the variability is (i.e., the spread) among observations. if the variability is small, all the values are close to the mean. if it is large, the values are not close to the mean. the significance of the standard deviation is that with normal (bell-shaped) distributions, the following empirical rules for the normal curve apply: -the interval -1 to 1: 68% -interval -2 to 2: 95% -interval -3 to 3: 99.7%

When reviewing the gram stain of a person with a wound infection, the IP sees gram-positive organisms in clusters. Which organism would this most likely represent? a. Streptococcus b. Enterococcus c. Corynebacterium d. Staphylococcus

D staphylococcus is a genus of gram-positive bacteria. under the microscope, they appear round (cocci) and form in grape-like clusters. Staphylococcus includes at least 40 species. Of these, nine have two subspecies and one has three subspecies. most are harmless and reside normally on the skin and mucous membranes of humans and other organisms. found worldwide, they are a small component of soil microbial flora

The current community-acquired pneumonia (CAP) national quality measures used in the US include all of the following except: a. Antibiotic timing (within 6 hours of arrival) b. Antibiotic selection c. blood cultures performed in the ED before antibiotics were administered d. ensure that all patients are screened for pneumococcal vaccination

D the core quality measures are a set of standards defined by the Joint Commission and the Centers for Medicaid and Medicaid Services (CMS) to create core measures for disease management for CAP. The measures create consistent evidence-based practice in facilities caring for CAP patients. CMS tracks hospitals' adherence to the measures and publishes the results. The core measures for CAP include: -oxygen assessment within 24 hours before or after arrival at the hospital -patients over 65 screened for pneumococcal vaccination and administration before discharge, if necessary -blood cultures performed in the ED before antibiotics were administered -antibiotic timing (within 6 hours of arrival) -antibiotic selection -adult smoking cessation advice and counseling provided -influenza vaccination

Obstacles for building a culture of patient safety in health care include all of the following except: a. assignment of blame on health care providers b. high staff turnover rates c. lack of resources for needed change d. placement of accountability on health care systems

D the creation, maintenance, and periodic measurement of a culture of safety are now health system regulatory requirements. attributes of a safety culture include placing a high priority on safety; allocating the appropriate resources, structure, and accountability to promote a culture of safety; encouraging and rewarding the identification, communication, and resolution of safety issues; and providing a structure and process to learn from mistakes. management has a set of responsibilities that include educating staff on event reporting, making continuous safety improvements, and identifying system flaws and potential corrective actions. managers must focus on the "how", not the "who" of an event, while underscoring individual accountability and responsibility

Which of the following is not part of the bundle practices to reduce VAP? a. keeping the head of the bed raised to 30 to 45 degrees elevation unless medically contraindicated b. Performing regular oral care one a ventilated patient c. taking sedation "vacations" to assess patients' ability to breathe on their own d. changing ventilator circuits every 48 hours

D the following best practices, often included in a ventilator bundle, can help prevent VAP: -keep the head of the patient's bed raised between 30 and 45 degrees unless other medical conditions do not allow this to occur -check the patient's ability to breathe on his or her own every day so that the patient can be taken off the ventilator as soon as possible -clean the patient's hands with soap and water or an alcohol-based hand rub before and after touching the patient or the ventilator -clean the inside of the patient's mouth on a regular basis

Seventy-five patients were admitted to the Meg-Surg ICU. Forty were on the surgical service and 35 were on the medical service. Fifteen patients developed a health care associated infection with MRSA. Nine of the patients with MRSA were on the surgical service. There were 230 patient days in the ICU for the surgical patients in January, and 325 patient days for medical patients. What was the incidence density of MRSA attack infection for patients on the surgical service? a. 29 infections per 1000 patient days b. 26 infections per 1000 patient days c. 19 infections per 100 patient days d. 39 infections per 1000 patients days

D the incidence rate is the number of new cases per population at risk in a given time period. when the denominator is the sum of the person-time at the at-risk population, it is also known as the incidence density rate or person-time incidence rate. the incidence-density rate for this scenario is 9 (new cases of MRSA) / 230 (total number of patient days) x 1000 = 39.13 (round to 39) infections per 1000 patient days

The Joint Commission standards for infection prevention and control include all of the following except: a. collaboration of representatives from relevant components and functions within the organization in the implementation of the program b. effective management of the infection prevention and control program c. minimizing the risk for development of a health-care associated infection (HAI) through an organization-wide infection prevention program d. specific staffing requirement of one infection preventionist (IP) for every 100 beds in the facility

D the joint commission lists five standards for infection prevention and control, which include minimizing the risk for development of an HAI through an organization-wide infection prevention program, identification of risk for the acquisition and transmission of infectious agents on an ongoing basis, effective management of the infection prevention and control program, collaboration of representatives from relevant components and functions within the organization in the implementation of the program, and allocation of adequate resources to the infection prevention and control programs. However, there is no specific staffing requirement

Which of the following statements is true when the prevalence of a disease is very low? a. the sensitivity of a diagnostic test is greatly increased b. the specificity of a diagnostic test is much greater c. the negative predictive value of a diagnostic test is very low d. the positive predictive value of a diagnostic test is lowered

D the measures of sensitivity and specificity describe how well the proposed screening test performs against a "gold standard" test. sensitivity and specificity are independent of prevalence of disease. the PPV describes the probability of having the disease given in a positive screening test result in the screened population. The NPV describes the probability of not having the disease given a negative screening test result in the screened population. PPV and NPV are disease prevalence dependent. Generally, a higher prevalence will increase the PPV and decrease the NPV

All of the following maternal infections would require withholding breast milk from the newborn except: a. breast abscess b. HIV c. untreated, active TB d. hepatitis C

D the only infections in which breast milk must be withheld from the newborn are: 1) presence of a breast abscess, 2) HSV lesion on breast, and 3) infection with HIV, West Nile virus, or human T-cell lymphotropic virus type I or II. If mastitis is present, breast-feeding can continue. if a breast abscess is present, the mother should pump the breast milk and discard it (until 24 hours after surgical drainage and appropriate antimicrobial therapy). women with open, active, untreated pulmonary TB cannot breast-feed because they are to have no direct contact with the newborn. however, breast milk can be pumped and given to the newborn, provided that the treatment the woman is receiving is not a contraindication for breast-feeing. currently, maternal HCV is not considered a contraindication for breast-feeding. the decision to breast-feed in the presence of maternal HCV must be an informed decision made by the woman in consultation with her health care provider.

The purpose of a root cause analysis is to: a. Determine which individual made an error so that the employee may be disciplined or terminated b. Review the basic processes that are in place and then turn that review over to a unit-specific team so that they can determine how they should modify their practices c. Provide a process that requires little time or training but allows employees to identify culpability after an adverse event d. Include participants from diverse areas of the organization to delve into the cause of an error or systems failure and identify changes in practice and/or policy that will prevent a repeat of that error or event

D the root cause analysis (RCA) process takes a retrospective look at adverse outcomes and determines what happened, why it happened, and what an organization can do to prevent the situation from recurring. it avoids individual blame, considers human factors engineering, and analyzes redesign for a safer system. identifies changes to a particular process or system that improves safety or reduces process error. can be time consuming, require training, expensive, deep dive

A pandemic differs from an epidemic in that: a. only one disease is involved b. it is usually vector borne c. there is a higher mortality rate d. several countries or continents are involved

D the term "pandemic" refers to an epidemic of disease spread over a wide geographical area across countries or continents

Ultrasonic cleaners are used for: a. Removing gross soil from an instrument b. High-level disinfection of instruments c. Sterilization of instruments d. Fine cleaning of instruments

D ultrasonic cleaning systems use high-frequency sound waves to remove particles from difficult to access areas such as lumens, joints, and crevices. meant for fine cleaning. before use, instruments should first undergo general cleaning to remove soil. not meant for final disinfection or sterilization

The IP is developing a seasonal influenza immunization promotion program and decides to survey some health care personnel to determine their knowledge and attitude about influenza vaccines. Several health care personnel state that they do not want to be immunized because they believe that the vaccine can give them the flu. What is the best response the IP can give to alleviate this fear? a. The symptoms of the flu from the vaccine are much milder than actually getting the flu, so they are better off being immunized b. There are no known reactions or side effects to the flu vaccine c. Any symptoms they experience are due to allergies to components of the vaccine, so they will not get the flu from the vaccine d. They might experience symptoms that are due to the immune response to the vaccine, but they cannot get the flu from the vaccine

D vaccination is the primary method for preventing influenza and its complications. Health care personnel compliance with annual influenza vaccination is an expected behavior to protect patients, staff, and families. There are two forms of vaccination available: the inactivated injectable vaccine and the live attenuated intranasal vaccine (LAIV). Inactivated influenza vaccine contains noninfectious viral components and cannot cause influenza. LAIV can cause mild symptoms such as nasal congestion, sore throat, and headache as a result of the immune response to the vaccine, but these are typically minor and short-lived.

In a study of whether Operating room A (OR A) is associated with a higher number of surgical site infections (SSIs) than Operating room B (OR B), the infection preventionist (IP) is testing whether: (Null hypothesis) Ho: OR A SSI rate = OR B SSI rate (Alternative hypothesis): Ha OR A SSI rate =/ OR B SSI rate The IP concludes the SSI rate in OR A is not equal to the SSI rate in OR B, but in reality, the two rates are equal. What type of statistical error was committed? a. No error has been committed b. Committed a Type I error c. Committed a Type II error d. Committed an error equal to B

If the IP concluded that the SSI rate in OR A is not equal to the SSI rate in OR B, then they reject the null hypothesis. However, in this case the null hypothesis was true, therefore, a Type I error was committed. This value is equal to alpha. type 1 error (false positive): reject null that is actually true type 2 error (false negative): fails to reject a null that is actually false

A patient was just admitted to a long-term care facility from the local hospital. The patient is being treated for psoriasis. The psoriasis does not appear to be responding to treatment, and 48 hours later, the infection preventionist (IP) receives a report that a CNA has developed an itchy rash. The patient's physician visits and determines that the patient has crusted scabies and not psoriasis. Another name for crusted scabies is:

Norwegian scabies Crusted scabies is a severe form of scabies that can occur in individuals who may be immunocompromised, elderly, disabled, or debilitated. It is also called Norwegian scabies. In immunodeficient individuals and in senile patients, infestation often appears as a generalized dermatitis more widely distributed than the burrows, with extensive scaling and sometimes vesiculation and crusting (Norwegian or crusted scabies); the usual severe itching may be reduced or absent

According to the CDC, which type of thermometer should be used in a vaccine storage unit? a. Fluid-filled biosafe liquid thermometer b. Infrared thermometer c. Chart recorder d. Probe in a glycol-filled bottle with an external monitoring device

The CDC recommends using a temperature probe in a bottle filled with a thermal buffer, like glycol, that connects to an external monitoring device. This allows for temperatures to be monitored without having to open the unit door. In addition, the CDC recommends the use of digital data loggers. The CDC does not recommend the use of fluid-filled biosafe liquid thermometers, infrared thermometers, or chart recorders.

What type of meningitis would be most consistent with the following cerebrospinal fluid report result? Glucose: Decreased Protein: Elevated WBC counts: 1000/mm3 a. bacterial b. viral c. fungal d. tuberculosis

a. bacterial look at table in notes. viral and fungal show similar trends but closed to normal ranges. when meningitis is suspected, it is common to collect a specimen of CSF. four basic components are considered: color and clarity, protein, glucose, and WBCs (including differential).

The primary immune response after exposure to a communicable disease pathogen or vaccine is production of: a. IgG b. IgM c. IgA d. IgC

b. IgM IgM is produced first in response to pathogen or vaccine. IgG antibodies develop a few weeks later. IgG antibodies are good indication on the convalescence period and generally mark the establishment of long-term immunity to the pathogen

Mode

the mode represents the observations that occur most frequently in a data set and determines the height and shape of a curve. Data sets may have more than one mode and can be bimodal or multimodal. small data sets may be nonmodal (ie there are no repeated values). the mode is most useful for describing qualitative data and is used for nominal data and bimodal distributions. it is the least stable of the three measures of central tendency.


संबंधित स्टडी सेट्स

Intro. To Business Management Mid-Term

View Set

Minds and Brains Final- Extensive

View Set

Pharmacology Exam 5 - Ear & Eye Disorders

View Set

Study Guide: Appendicular Skeleton Session Questions

View Set

L'Oreal Superday - Technical Questions

View Set